pharmacology final1

Pataasin ang iyong marka sa homework at exams ngayon gamit ang Quizwiz!

258. Which of the following most likely may cause hepatotoxicity if overused?

Acetaminophen (paracetamol)

259. Xerostomia can be caused especially frequently by?

Agents with cholinolytic activity

268. Which one of the following drugs is most likely to block K+ channels in the heart responsible for cardiac repolarization, and also blocks calcium channels in the AV node?

Amiodarone

192. Which from the following statements concerning intravenous sedation is false: A. Can be excellently titrated B. Patient who is treated with i.v. sedation does not require to be escorted home C. During i.v. sedation heart rate, blood pressure, and respiration should be monitored D. Benzodiazepines are recommended for i.v. sedation E. Intravenous sedation is fairly accepted by patients

B

280. Drug Biotransformation which is true: A. Each drug undergoes hepatic biotransformation B. Products may be more biologically active as a drug C. Occurs exclusively in the liver D. Products are already less biologically active as a drug E. Biotransformation products are never toxic

B

72. A 56-year-old male presents with loose stools containing blood and mucous, abdominal pain and cramps. Diagnosis was pseudomembranous colitis. He was put on antibiotics a week earlier for osteomyelitis. A. Tetracycline B. Vancomycin C. Gentamicin D. Ketoconazole E. Clindamycin

B

569. Amiodarone: 1. is effective in atrial fibrillation 2. is effective in ventricular tachycardia 3. is inotropic positive agent 4. increases blood pressure 5. prolongs phase 3 of the electrochemical potential A. 1,3,4 B. 1,2,3 C. 1,2 D. 4,5 E. 1,2,5

B. 1,2,3

519. Which of the following substances (metabolite) causes urinary bladder bleeding and damage during cyclophosphamide pharmacotherapy? A. Dexrazoxane B. Acrolein C. Aldophosphamide D. MESNA E. Nitrogen mustard

B. Acrolein

444. A 45-years-old patient suffers from burning and itching of the lips at the right side of intraoral pain. He has difficulties in swallowing. His complaints appeared post exposure to the sun and it was the fifth time in his life. On examination vesicles, ulcerations on his lips and intraorally appreard. Which antiviral agent is the first choice for the patient? A. Vancomycin B. Acyclovir C. Interferon gamma D. Amantadine E. Ribavirin

B. Acyclovir

107. A 34-year old female patient is treated for supraventricular tachycardia with a drug that prolongs atrioventricular conduction. This drug is: A. Milrinone B. Adenosine C. Nitroglycerine D. Lidocaine E. Hydralazine

B. Adenosine

357. The sudden withdrawal of cortisol after a prolonged therapy with this drug may result in: A. Diabetes mellitus B. Adrenal insufficiency C. Moon face D. Osteoporosis E. Hypertension

B. Adrenal insufficiency

7. The proper dose of fluoride supplement for children is dependent on: A. Children's diet B. Age and fluoride concentration of the water supply C. Liver function D. Route of administration E. Type of caries preexisting fluoride supplementation

B. Age and fluoride concentration of the water supply

303. The principal aims of the clozapine use for schizophrenia are to minimize the symptoms and prevent exacerbations. It produces less risk for extrapyramidal effects but can cause fatal: A. Akathisia B. Agranulocytosis C. Alogia D. Anhedonia E. Avolition

B. Agranulocytosis

181. The principal goals with use of clozapine for schizophrenia are to minimize symptoms and prevent exacerbations. It produces less risk for extrapyramidal effects but can cause fatal: A. Akathisia B. Agranulocytosis C. Alogia D. Anhedonia E. Avolution

B. Agranulocytosis (risk for blood agranulocytosis with clozapine)

359. A 33 years old woman has sporadic attacks of intense anxiety with marked physical symptoms, including hyperventilation, tachycardia and sweating. If she is diagnosed as suffering a panic disorder, the most appropriate drug to use is: A. Phenobarbital B. Alprazolam C. Acebutolol D. Chloral hydrate E. Zaleplon

B. Alprazolam

583. A 23 year old woman has sporadic? attacks of intense anxiety with ... physical symptoms, including hyperventilation, tachycardia and sweating. If she is diagnosed as suffering an anxiety/panic (?) disorder, the most appropriate drug to use is: A. Fluoxetine B. Alprazolam C. Hydro.... D. Digoxin

B. Alprazolam

377. Hyperkalemia, hyponatremia and metabolic acidosis were revealed in your patient treated before with a certain diuretic. The most probably it was: A. Furosemide B. Amiloride C. Acetazolamide D. Bumetanide E. Hydrochlorothiazide

B. Amiloride

378. Which of the following would be most unacceptable to use in the treatment of a hospital-acquired infection caused by P. aeruginosa? A. Ceftazidime B. Amoxicillin C. Aztreonam D. Amikacin E. Piperacillin + tazobactam

B. Amoxicillin

379. A 37-year old woman with HIV infection suffers from concomitant fungal pneumonia. Which agent from the following is best choice for her? A. Morpholines B. Amphotericin B C. Griseofulvin D. Cilcopirox E. Flucytosine

B. Amphotericin B

503. Which of these drugs has the longest half life: A. Erythromycin - first order elimination rate constant ke=0.02 hr-1 B. Amphotericin B - first order elimination rate constant ke=0.01 hr-1 C. Methicillin - first order elimination rate constant ke=0.93 hr-1 D. Vancomycin - first order elimination rate constant ke=0.10 hr-1 E. Amikacin - first order elimination rate constant ke=0.03 hr-1

B. Amphotericin B - first order elimination rate constant ke=0.01 hr-1

45. The pharmacological effects of aspirin comprise all of the following EXCEPT: A. Analgesic effect B. Antiallergic effect C. Antipyretic effect D. Anticoagulant effect E. Anti-inflammatory effect

B. Antiallergic effect

484. Xerostomia can be caused especially frequently by: A. NSAIDs B. Antihistamines C. Dihydropyridines D. Insulin E. Beta-blockers

B. Antihistamines

402. A boy is treated with an agent that activates nicotinic acetylcholine receptors inducing ultimately paralysis of neuromuscular synapsis in a helminth. Which organism is most likely infecting this patient? A. Taenia solium B. Ascaris lumbricoides C. Schistosoma mansoni D. Onchocerca volvulus E. Trichinella spiralis

B. Ascaris lumbricoides

43. Which from the following agents has to be avoided in therapy of pain in children and teenagers with suspected viral infection: A. Acetaminophen B. Aspirin C. Nabumetone D. Ibuprofen E. Ketoprofen

B. Aspirin

453. Which of the following is true concerning the indication for glucocorticosteroids: A. Oral candidiasis B. Autoimmune diseases C. Hypertension D. Diabetes E. Severe infections

B. Autoimmune diseases

508. Which of the following drugs is useful in the treatment of postoperative urinary retention? A. Alfuzosin B. Bethanechol C. Galantamine D. Atropine E. Hexamethonium

B. Bethanechol - cholinomimetic (direct acting)

312. The treatment of neuropathic pain includes all of the following methods except for: A. Opioids B. Bisphosphonates C. Gabapentin D. Amitriptyline E. Local anesthetics

B. Bisphosphonates

11. The most appropriate drug for the treatment of generalized anxiety disorder (GAD) is: A. Secobarbital B. Buspirone C. Midazolam D. Zolpidem E. Triazolam

B. Buspirone

483. The most appropriate drug for treatment of generalized anxiety disorder (GAD) is: A. Amoxicillin B. Buspirone C. Amitriptyline D. Ibuprofen E. Haloperidol

B. Buspirone

483. The most appropriate drug for treatment of generalized anxiety disorder (GAD) is: A. Secobarbital B. Buspirone C. Midazolam D. Zolpidem E. Triazolam

B. Buspirone

457. Which of the following is true for antiarrhythmic agents: A. Their therapeutic index for most drugs extremely high B. Can cause arrhythmias C. Are use only at hospital conditions D. Amlodipine is an example of one among most commonly used antiarrhythmics E. Alpha-2-agonists are commonly used as antiarrhythmics

B. Can cause arrhythmias (proarrhythmogenic effect)

455. In P.B. 56-year old patient treated with furosemide was on his routine visit to his dentist. Suddenly he felt worse. Which of the following could be the reason of this patient's status worsening: A. Suddenly increased blood pressure B. Cardiac arrhythmia C. Diarrhea D. Vomiting E. Increased salivation

B. Cardiac arrhythmia

331. A 72-year old patient presents to the clinic with untreated hypertension. Past medical history includes coronary artery disease, myocardial infarction, cardiac insufficiency. There is a strong indication for initiating a beta-blocker in this patient. Which one would you select? A. Esmolol B. Carvedilol C. Pindolol D. Propranolol E. Nadolol

B. Carvedilol

165. 45-year-old man presents to his family doctor complaining of progressive weakness, cough and fever (37.5 C). Atypical community acquired pneumonia caused by Chlamydia sp. has been diagnosed. Which of the following antibiotics should be not used in the therapy of this type of pneumonia? A. Clarithromycin B. Cefuroxime C. Trimethoprim D. Doxycycline E. Ciprofloxacin

B. Cefuroxime - cephalosporin, no effect on chlamydia species

239. In a patient with a history of gastric ulcer dental procedure associated with moderate pain was performed. Which of the following can you recommend as the most effective and safe pain killer in this patient? A. Morphine as a long acting oral tablet, 4-6 times a day B. Celecoxib, orally, 100 mg 2 times a day C. Aspirin 50 mg, 4-6 times a day D. Paracetamol (Acetaminophen), i.v. 500 mg, 2 times a day E. Aspirin 500 mg, 4-6 times a day

B. Celecoxib, orally, 100 mg 2 times a day

552. Salicylates cannot be used for:

B. Children younger than 12 who have symptoms of viral infection

67. Miosis, salivation, lacrimation, increased bronchial secretions, wheezing, bradycardia, diarrhea and involuntary micturition are effects of: A. Beta-adrenergic antagonists B. Cholinomimetics C. Cholinergic receptors antagonists D. Nicotinic receptor agonists E. Antimuscarinic drugs

B. Cholinomimetics

166. Which of the following drugs often exerts a clinically significant dose dependent neurotoxic effect? A. Bleomycin B. Cisplatin C. Vincristine D. Doxorubicin E. Methotrexate

B. Cisplatin - crosslink DNA - dose dependent neurotoxicity

411. A woman developed bleeding that required a reduction of her warfarin dose while being treated for an intra-abdominal anaerobic infection. Which antibiotic was most likely responsible for this drug interaction? A. Aztreonam B. Clindamycin C. Gentamicin D. Doxycycline E. Amikacin

B. Clindamycin

525. In a patient with the very first symptoms of tardive dyskinesia, the best choice of next drug is: A. Quetiapine B. Clozapine C. Haloperidol D. Olanzapine E. Risperidone

B. Clozapine

306. All of the following are capable of inducing a signal transduction process EXCEPT for: A. Combination of an agonist with its receptor B. Combination of an antagonist with its receptor C. Combination of a neurotransmitter with its receptor D. Combination of a hormone with its receptor E. Combination of a drug with its receptor

B. Combination of an antagonist with its receptor

499. Which of the following best describes a drug that binds to alpha adrenergic receptors, has no intrinsic activity, and can be displaced by epinephrine (agonist)? A. Chemical antagonist B. Competitive antagonist C. Irreversible antagonist D. Partial agonist E. Functional antagonist

B. Competitive antagonist

119. A 40-year-old male was treated for alcohol abuse and malnutrition related megaloblastic anemia. In addition, he was being given drug X which resulted in remission of anemia. Vitamin B 12 was discontinued. Drug X in a person with vitamin B 12 deficiency can produce: A. Correction of vitamin B 12 absorption B. Correction of neurological abnormalities C. Correction of vitamin B 12 plasma level D. Correction of intrinsic factor synthesis E. Correction of tetrahydrofolate plasma level

B. Correction of neurological abnormalities

40. Which one of the following poison is improperly matched with the indicated treatment of antidote? A. Arsenic : dimercaprol (BAL) B. Cyanide : calcium glucaonate C. Iron : Deferoxamine D. Lead : calcium disodium edetate (EDTA) E. Copper : penicillamine

B. Cyanide : calcium glucaonate

336. Which of the following combination of treatment is most likely to be effective in the treatment of leprosy? A. Pyrimethamine and Sulfonamide B. Dapsone and Rifampin and Clofazimine C. Albendazole and Corticosteroids D. Praziquantel and Corticosteroids E. Metronidazole and Doxycycline

B. Dapsone and Rifampin and Clofazimine

133. All of the following symptoms are indicative of marijuana intoxication, EXCEPT: A. Panic reaction B. Depression C. Drowsiness D. Alteration in time perception E. Alteration in vision

B. Depression

578. Which answer about transdermal drug delivery systems is false: A. Can cause skin irritation B. Dosage forms for local administration of drug C. Ensure gradual and even absorption of drug D. Provide increased bioavailability of drug by elimination of first-pass effect E. Can be used to deliver drugs for pain relief

B. Dosage forms for local administration of drug

518. Which answer about transdermal drug delivery system is false: A. Can cause skin irritation B. Dosage forms for local administration of the drug C. Ensure the gradual and even absorption of the drug D. Provide increased bioavailability of the drug by elimination of the first-pass effect E. Can be used to deliver drugs for pain relief

B. Dosage forms for local administration of the drug - dosage form for systemic administration

439. A 33-year old pregnant woman requires an antibiotic. Which of the following agents is mandatory contraindicated? A. Clindamycin B. Doxycycline C. Erythromycin D. Cefazolin E. Amoxicillin

B. Doxycycline

418. A progestin is included in regimens for HRT to prevent certain adverse effects. Which one? A. Breast cancer B. Endometrial cancer C. Myocardial infarction D. Stroke E. Elevated cholesterol levels

B. Endometrial cancer

446. During his visit in dental office, when injected with mepivacaine, a 25-years old otherwise healthy men felt worse. He was confused pale, his skin cold and wet. His respiratory rate 23/min, heart rate was 120/min, BP=70/30 mm hg. Which of the following route of epinephrine administration is recommended for him? A. Epinephrine i.v. B. Epinephrine i.m. C. Epinephrine intratrathechally D. Epinephrine in inhalation E. Epinephrine intraosseously

B. Epinephrine i.m.

537. If after childbirth, the uterus does not return to its normal size (subinvolution of the uterus), it can be treated with: A. Minoxidil B. Ergometrine C. Dinoprostone D. Fenoterol E. Magnesium sulfate

B. Ergometrine

551. In immunocompromised (patient with immunosuppression) patient suffers from candidiasis of the whole GI mucosa. Which antibiotic will be helpful for him? A. Gentamicin B. Fluconazole C. Ciprofloxacin D. In...de? E. Nystatin

B. Fluconazole

517. To optimize drug therapy, it is necessary to know in what phase of the cell cycle antineoplastic agents are effective. Which one of the following agents is cytotoxic only to cells in the S-phase of the cycle? A. Bleomycin B. Fluorouracil C. Cyclophosphamide D. Cisplatin E. Carmustine

B. Fluorouracil - antimetabolites (cell-cycle specific, S-phase)

62. A 45-year-old patient was treated within 6-month with a certain hormonal agent. Osteoporosis, hypertension and depressive psychosis were observed as results of therapy. These symptoms are adverse effects of: A. Estrogen B. Glucocorticosteroid C. Thyroid hormone D. Somatostatin E. Insulin

B. Glucocorticosteroid

82. B.T. is a 54-year-old investment banker and his chief complaint is abdominal pain. He noted first the pain about six week ago when it awoke him from sleep. Food and antacids would temporarily relieve the pain, but it nevertheless persisted. The diagnostic evaluation of B.T. revealed a duodenal ulcer that was visualized by endoscopy. B.T.`s other medical problem includes tennis elbow which he treats with aspirin 600 mg four times/day. Drug X in one dose at bedtime was prescribed for B.T. What is the mechanism of action of drug X assuming that this drug may decrease volume of gastric juice: A. Inhibition of H+/K+ ATP-ase B. H2-receptor antagonism C. M1-receptor antagonism D. Hydrochloric acid neutralization

B. H2-receptor antagonism

507. In a patient with severe psychotic symptoms with intense hallucinations, agitation, aggressive behavior, the best drug of the first choice will be: A. Quetiapine B. Haloperidol C. Clozapine D. Risperidone E. Olanzapine

B. Haloperidol - First generation Antipsychotic (high potency)

465. A patient takes indapamide every day. It allows us to conclude that he is most likely treated because of: A. Atrio-ventricular block B. Hypertension C. Heart failure D. Hypercholesterolemia E. Supraventricular arrhythmia

B. Hypertension

193. The pharmacological effects of nitrates comprise: A. Bradycardia B. Increased venous pooling C. Contraction of veins D. Direct chronotropic positive effect E. Direct dromotropic positive effect

B. Increased venous pooling

451. Indicate the incorrect pair: A. Metoprolol - B1-adrenolytic B. Indapamide - Ca-channel antagonist C. Ramipril - ACE inhibitor D. Doxazosin - a1-selective adrenolytic E. Clonidine - a2-agonist

B. Indapamide - Ca-channel antagonist

216. Local anesthetics are less infected tissues because: A. They are rapidly absorbed from the site of injection into blood B. Infected tissues have a low extracellular pH, so a very low fraction of non ionized drug can diffuse into the nerve cell C. They are degraded in acidic environment D. There is desensitization of receptors in sodium channels in nerve cells E. Accumulation of acid metabolites changes pK a of a local anesthetic agent

B. Infected tissues have a low extracellular pH, so a very low fraction of non ionized drug can diffuse into the nerve cell

299. A 65 year old obese man suffering from primary hypertension and type 2 diabetes eagerly participates in hearty business feasts which include meat, alcoholic beverages and sophisticated cheeses. One day, the man came to his doctor complaining that at night he experienced severe pain in the left big toe with redness and erythema. Identify the mechanism of action of a drug recommended by the physician for rapid relief of symptoms. A. Inhibition of phospholipase A2 B. Inhibition of COX-1 and COX-2 C. TNF- alpha receptor binding D. Inhibition of H1 receptor

B. Inhibition of COX-1 and COX-2 - NSAID

435. Which of the following drug interaction mechanisms is most likely to lead to sustained elevations of plasma drug concentrations and drug toxicity? A. Induction of CYP2C19 B. Inhibition of CYP3A4 C. Increased drug binding to plasma albumin-binding sites D. Induction of the P-glycoprotein carrier protein E. Acceleration of gastric emptying by a "prokinetic" drug

B. Inhibition of CYP3A4

403. A patient with shingles receives an antiviral drug. Which antiviral action is exerted by this agent? A. Blockade of purine biosynthesis B. Inhibition of DNA-polymerase C. Inhibition of viral entry D. DNA chain termination E. Prevention of viral maturation

B. Inhibition of DNA-polymerase

342. The long term beneficial effect of ACE-Is in the therapy of chronic heart failure are associated with: A. Inhibition of AT1-receptor stimulation B. Inhibition of angiotensin II-related left ventricle remodeling C. Inhibition of renal potassium reabsorption D. Increase in left ventricle contractility E. Sodium and Water excretion

B. Inhibition of angiotensin II-related left ventricle remodeling

212. A 6-year-old boy presented to his pediatrician with fever (101.3°F, 38.5°C) and sharp pain in his left ear. On physical examination, the left tympanic membrane was red, opaque, and bulging. Amoxicillin was prescribed, but 3 days later, the symptoms were not reduced. The pediatrician decided to modify the therapy and prescribed amoxicillin/potassium clavulanate. Which of the following best explains the advantage of adding potassium clavulanate to amoxicillin? A. Extended antibacterial spectrum B. Inhibition of bacterial inactivation of amoxicillin C. Inhibition of renal secretion of amoxicillin D. Increased amoxicillin entry into bacteria E. Decreased amoxicillin allergenicity

B. Inhibition of bacterial inactivation of amoxicillin

288. Sildenafil: A. Is administered by intracavernosal injection B. Inhibits calcium channels C. Can cause sudden hypotonia D. inhibits potassium channels E. Can cause hypertension

B. Inhibits calcium channels - Pulmonary Hypertension: sildenafil, bozantram, epoprostentol

127. A jaundiced one-day-old premature infant with an elevated free bilirubin level is seen in the premature-baby nursery. The mother had received an antibiotic for urinary tract infection (UTI) one week before delivery. You suspect that she had been treated with an agent that acts by the following mechanism: A. Enhances synthesis of bilirubin B. Inhibits folic acid synthesis C. Inhibits of bilirubin degradation D. Inhibits of urinary excretion of bilirubin E. Inhibits gyrase activity

B. Inhibits folic acid synthesis

447. Which of the following is classified as belonging to the tyrosine kinase family of receptors? A. GABAa receptor B. Insulin receptor C. B-adrenergic receptor D. Nicotinic II receptor E. Hydrocortisone receptor

B. Insulin receptor

553. Buprenorphine can precipitate abstinence syndrome to an opioid-dependent person because of one of the following statements: A. It is a potent k-receptor antagonist B. It is a strong partial agonist at mu opioid receptor C. It has a fewer psychotomimetics effect than pentazocine D. It fails to mimic the effect of morphine in drug-free patients E. It may interact with delta receptor

B. It is a strong partial agonist at mu opioid receptor

314. Which of the following drugs metabolized by cytochrome P450 can inhibit the clearance of other drugs: A. Carbamazepine B. Ketoconazole C. Phenobarbital D. Phenytoin E. Rifampicin

B. Ketoconazole

539. During 6-hour knee surgery, the anesthetist probably uses a slow infusion using a catheter with: A. Bupivacaine B. Lidocaine + adrenaline C. Tetracaine D. Benzocaine E. Adrenaline

B. Lidocaine + adrenaline

237. A 56-year-old woman, who had been suffering from atrial fibrillation for 3 years, was in the dentist's office for the application of an implant. The dentist was about to initiate local anesthesia by injecting a local anesthetic close to the mandibular nerve. Which of the following local anesthetic preparations would be most appropriate for this patient? A. Benzocaine alone B. Lidocaine alone C. Bupivacaine plus epinephrine D. Lidocaine plus epinephrine E. Benzocaine plus epinephrine

B. Lidocaine alone

8. Which of the following is true concerning bupivacaine: A. Rapid onset of action B. Long duration of action C. Indicated for intravenous regional anesthesia D. Ester-like structure E. Epinephrine reduces bupivacaine toxicity

B. Long duration of action

83. A.D. is a 22-year old medical student who has been suffering from excessive sweating, palpitations and diarrhea because of fear before the exam. Which of the following agents might be the most useful in alleviation of stress related diarrhea? A. Bisacodyl B. Loperamide C. Metoclopramide D. Sodium docusate E. Hyoscine hydrobromide

B. Loperamide

541. Which of the following benzodiazepines is the first drug at the Emergency Department in case of seizures: A. Diazepam B. Lorazepam C. Midazolam D. Alprazolam E. Oxazepam

B. Lorazepam

151. A 47-year-old man complained of severe pain for 3 days after-surgery to remove a colon cancer. Physical examination and x-ray confirmed the diagnosis of postoperative adynamic ileus. Analgesic treatment was prescribed. Which of the following analgesic drugs would be contraindicated for this patient? A. Acetaminophen B. Morphine C. Ketorolac D. Indomethacin E. Ibuprofen

B. Morphine - contraindicated constipation make the condition worse give Lidocaine instead to recover bowel function

426. Which of the following actions of epinephrine would be antagonized by alpha-1-selective adrenolytic but not by beta-antagonist? A. Increase in heart rate B. Mydriasis C. Release of renin D. Bronchiolar dilation E. Glycogenolysis

B. Mydriasis

14. Which of the following sentences concerning pain control is false: A. Long-acting anesthetics are recommended for the perioperative administration B. Opioids are contraindicated in combination with NSAIDs C. For non-opioid analgesics the oral route of administration is only possible D. Respiratory depression could be the result of opioids use E. Neuropathic pain require anticonvulsants or tricyclic antidepressants use to be sufficiently controlled

B. Opioids are contraindicated in combination with NSAIDs

168. Which of the following mechanism is most frequently responsible for the resistance of cancer cells classic cytostatic (cytotoxic agents)? A. decreased drug transportation into the cell B. P-glycoprotein increased expression C. Inhibition of conversion the pro-drug into its active metabolite D. Accelerated drug biotransformation E. Accelerated drug excretion

B. P-glycoprotein increased expression - MDR1 drug efflux pump (most common)

219. Which of the following is not used for therapy of drug induced oral ulcerations and mucosistis? A. Hydroxypropyl cellulose B. Pilocarpine C. Benzocaine D. Normal saline E. Kaolin solution

B. Pilocarpine

3. A 48-year-old woman took therapy sublingual nitroglycerin. Which of the following adverse effects would be most likely to occur in this patient? A. Methemoglobinemia B. Postural hypotension C. Venous thrombosis D. Diarrhea E. Reflex bradycardia

B. Postural hypotension

74. The drug of choice in treatment of most tapeworm infections is: A. Niclosamide B. Praziquantel C. Nifurtinox D. Metronidazole E. Sulfones

B. Praziquantel

488. Which antimalarial drug causes a dose-dependant toxic state that includes flushed and sweaty skin, dizziness, nausea, diarrhea, tinnitus, blurred vision and impaired hearing? A. Primaquine B. Quinine C. Sulfadoxine D. Mefloquine E. Pyrimethamine

B. Quinine

602. Which of the following drugs should you use in the prevention of the stress-related gastric ulcer: A. Calcium containing antacids B. Ranitidine C. Misoprostol D. Magnesium containing antacids E. Colloidal bismuth compounds

B. Ranitidine - PPI

478. Fluoxetine: A. Selectively blocks neuronal uptake of noradrenaline B. Selectively blocks neuronal uptake of serotonin C. Is more cardiotoxic than amitriptyline D. Is used for epilepsy treatment E. Is used for schizophrenia treatment

B. Selectively blocks neuronal uptake of serotonin

180. A.B. a 30-years-old women with diabetes type 1 hypoglycaemia (PG- 65 mg/dl-3.6 mmol/l) appeared. She is unconscious. Indicate the first choice therapy for her: A. She should eat chocolate snack B. She should inject with 1 mg of glucagon i.v. C. She should drink a cup of tea with 2 spoons of sugar D. She should infused with 50% glucose s.c. E. She should be infused with 5% glucose i.v.

B. She should inject with 1 mg of glucagon i.v.

577. Mechanisms of hereditary genetic variability, resulting in a different response to drug action, is very often coupled with a single nucleotide polymorphism (SNPs). Which of the below listed process is not involved with SNPs: A. Variable number of tandem repeats B. Silencing C. Duplication D. Insertion E. Deletion

B. Silencing

521. Which of the following will not be blocked by atropine and scopolamine? A. Miosis B. Skeletal muscle contraction C. Bradycardia D. Bronchoconstriction E. Salivary secretion

B. Skeletal muscle contraction

99. T.D. has been receiving oral anticoagulant treatment for his pulmonary embolism that occurred 2 months ago. This treatment is monitored for bleeding episodes and for the occurrence of: A. Osteoporosis B. Skin necrosis C. Hypersensitivity reactions D. All of the above E. None of the above

B. Skin necrosis

636. Indicate which of the following statements concerning possible results of HMG-CoA reductase inhibition, is true: A. The synthesis of LDL receptors in hepatocytes increases the presence of statins B. Statins, by reducing the HMG-CoA activity, reduce the oxidation of LDL particles C. Fibrates, by reducing the HMG-CoA activity, reduce the oxidation of LDL particles D. Ezetimibe, by reducing the HMG-CoA activity, reduce the oxidation of LDL particles E. The synthesis of VLDL receptors in hepatocytes increases in the presence of statins

B. Statins, by reducing the HMG-CoA activity, reduce the oxidation of LDL particles - pravastatin/simvastatin

220. Indirect effects of cancer chemotherapy in oral cavity comprise all the following, except: A. Salivary gland dysfunction B. Temporomandibular dysfunction C. Infections of mucosa D. Periodonatal infection E. Pulpal infection F. Dental caries

B. Temporomandibular dysfunction

456. Hypercalcemia is the possible adverse effect of: A. Loop diuretics B. Thiazides C. Spironolactone D. Sodium channels inhibitors E. Carbonic anhydrase inhibitors

B. Thiazides

123. A 24-year-old AIDS patient is interested in starting chemoprophylaxis for Pneumocystis pneumonia (PCP) and cerebral toxoplasmosis. He has no drug allergies. Which of the following prophylactic agents is appropriate for the prevention of both PCP and cerebral toxoplasmosis? A. nitrofurantoin - damages bacterial DNA, used in UTI B. trimethoprim - sulfamethoxazole C. norfloxacin - fluoroquinolone, block DNA gyrase and topo II D. methenamine - UTI, forms formaldehyde and ammonia E. nalidixic acid - quinolone, inh. Topoisomerase II, IV and gyrase

B. Trimethoprim - sulfamethoxazole - first choice in prophylaxis and treatment of pneumocystis pneumonia

69. Which of the following antiemetics acts through blocking serotonergic receptors? A. Promethazine B. Tropisetron C. Hyoscine D. Metoclopramide E. Dimenhydrinate

B. Tropisetron

573. A 45-year-old female suffers from tachycardia in the course of heart failure. Her therapeutic schedule is as follows: ramipril, furosemide, digoxin, carvedilol, spironolactone. Her BP is 110/70 mm Hg, HR-110/min, ECG-sinus tachycardia. Indicate the proper pharmacological correction of her tachycardia, suitable just for her: A. Use higher dose of beta-blocker B. Use eplerenone instead of spironolactone C. Add ivabradine to therapeutic regimen D. Add verapamil to therapeutic regimen E. Discontinue digoxin

B. Use eplerenone instead of spironolactone

480. Which of the following drugs is an anti-epileptic agent? A. Rifampin B. Valproate C. Isoniazid D. Zopiclone E. Haloperidol

B. Valproate

105. Patient RT received a chemotherapeutic drug for his leukemia which produced paresthesia and numbness in the legs. This drug could have been: A. Methotrexate B. Vincristine C. Fluorouracil D. Bleomycin E. Doxorubicin

B. Vincristine (hodgkins lymphoma)

341. A 55-year old alcoholic is treated with carbohydrate infusion (5% glucose solution). In this patient nystagmus, ophthalmoplegia, cerebral ataxia and confabulatory psychosis has rapidly developed: A. Vitamin A B. Vitamin B1 (Thiamine) C. Vitamin B2 (riboflavin) D. Vitamin B6 (pyridoxine) E. Niacin F. Vitamin C G. Vitamin D

B. Vitamin B1 (Thiamine)

637. 65 yr pt receives lovastatin for hypercholesterolemia. During treatment w/ lovastatin it is important for his physician to monitor plasma % (plasma count) of: A. glucose B. alanine/asparagine aminotransferase and creatine phosphokinase C. platelets count D. neutrophil count E. uric acid and creatine

B. alanine/asparagine aminotransferase and creatine phosphokinase

21. A 33-years-old woman has sporadic attacks of intense anxiety with marked physical symptoms, including hyperventilation, tachycardia and sweating. If she is diagnosed as suffering a panic disorder, the most appropriate drug to use is: A. phenobarbital B. alprazolam C. acebutolol D. chloral hydrate E. zaleplon

B. alprazolam

23. The mechanism of action of anti-neoplasmatic alkaloids (vincristine, vinblastine) comprises: A. binding to dihydrofolate reductase B. binding to microtubular protein C. altering fluidity and ion transport in membrane D. blocking estrogen receptors E. alkylating DNA in the N7 position of guanine

B. binding to microtubular protein

497. The mechanism of action of antineoplastic alkaloids (vincristine, vinblastine) comprises: A. binding to dihydrofolate reductase B. binding to microtubules protein C. altering fluidity and ion transport in membrane D. blocking estrogen receptors E. alkylating DNA in the N7 position of guanine

B. binding to microtubules protein (tubulin)

571. Which of the following statements concerning verapamil is false: A. blocks slow inward calcium current B. can cause diarrhea C. slows rate of AV-conduction D. can cause sinus bradycardia E. causes inotropic negative effect

B. can cause diarrhea - causes constipation

616. Ipratropium bromide: A. is administered intravenously B. causes bronchodilation because of its antagonistic effects at the cholinergic M2/M3 receptors C. has a more rapid onset of bronchodilation than beta 2 agonists D. A and B are true E. B and C are true

B. causes bronchodilation because of its antagonistic effects at the cholinergic M2/M3 receptors

150. Mercaptopurine toxicity in some patient approximately 5% may be significantly increased as a result of genetically determined metabolic disorder (pharmacogenetic syndrome) consisting of: A. Deficiency or lack of dihydropyrimidine dehydrogenase B. deficiency or lack of thiopurine methyltransferase TPMT C. deficiency or lack of xanthine oxidase D. deficiency or lack of folate dehydrogenase E. deficiency or lack of asparaginase

B. deficiency or lack of thiopurine methyltransferase TPMT - Asian population lack this enzyme, severe side effect are seen

26. Which of the following agents induce the most frequently Quincke's edema: A. losartan B. enalapril C. doxazosin D. omeprazole E. ranitidine

B. enalapril

619. Administration of mepivacaine resulted in anaphylactic shock development. Which drugs should be considered in the therapy? A. norepinephrine 1 mg i.v., methyloprednisolone 40-125 mg i.v., clemastine 2 mg i.v., ranitidine 50 mg i.v. B. epinephrine 1 mg i.v., methyloprednisolone 40-125 mg i.v., clemastine 2 mg i.v., ranitidine 50 mg i.v. C. epinephrine 1 mg i.v., methyloprednisolone 40-125 mg i.v., dopamine 2 mg i.v., ranitidine 50 mg i.v. D. epinephrine 1 mg i.v., fludrocortisone 40-125 mg i.v., clemastine 2 mg i.v., ranitidine 50 mg i.v. E. epinephrine 1 mg i.v., methyloprednisolone 40-125 mg i.v., clemastine 2 mg i.v., omeprazole 50 mg i.v

B. epinephrine 1 mg i.v., methyloprednisolone 40-125 mg i.v., clemastine 2 mg i.v., ranitidine 50 mg i.v.

17. Local anaesthetics are less effective in infected tissues because: A. they are rapidly absorbed from the site of injection into blood B. infected tissues have a low extracellular pH, so a very low fraction of non-ionized drug can diffuse into the nerve cell C. they are degraded in acidic environment D. there is desensitisation of receptors in sodium channels in nerve cells E. accumulation of acid metabolites changes pKa of a local anaesthetic agent

B. infected tissues have a low extracellular pH, so a very low fraction of non-ionized drug can diffuse into the nerve cell

148. 18-year old boy for suicide purposes swallowed number of unknown tablets. After 1 hour fusible vomiting followed by bloody diarrhea. These symptoms then muted for several hours. After this time, the boy was found by his grandmother in a coma. Rescuer stated pale coating, kussmaul breath, hypovolemic shock. What substance could these tablets contain? A. verapamil B. iron compounds C. tricyclic antidepressant D. benzodiazepine E. gliclazide

B. iron compounds

68. Lactulose: A. is an effective prokinetic agent B. is a laxative C. is and H2 blocker D. is a M1 blocker E. none of the above is true

B. is a laxative

611. Bromocriptine: A. stimulates lactation B. is used to treat hyperprolactinemia C. is a alpha 2 receptor agonist D. commonly causes diarrhea E. is effective in reducing nausea

B. is used to treat hyperprolactinemia

307. The best choice for chronic therapy of patient with coronary heart disease and liver cirrhosis will be: A. nitroglycerin given orally B. isosorbide mononitrate given orally C. isosorbide dinitrate given sublingually D. isosorbide dinitrate given orally E. nitroglycerine given sublingually

B. isosorbide mononitrate given orally (daily use)

265. Potent inhibitor of cytochrome p450? A. terbinafine B. ketoconazole C. ciclopirox D. nystatin E. polimyxin F. praziquantel

B. ketoconazole

291. Which of the following drugs should be prescribed to alleviate persistent dyspepsia, bloating and nausea—usual symptoms of gastroesophageal reflux, that has been diagnosed in a female 44 yrs old pt? A. loperamide B. metoclopramide C. bisacodyl D. hydroxyzine E. amiodarone

B. metoclopramide

347. Pharmacological antagonists: A. cause receptor down regulation B. prevent binding of other molecules to the receptor by their binding to the receptor C. example: atropine and acetylcholine D. A and B E. B and C

B. prevent binding of other molecules to the receptor by their binding to the receptor

292. A 49 yr pt is suffering from duodenal ulcer disease. Additionally the infection of Helicobacter pylori was diagnosed. The first-line therapy in this pt should include: A. ranitidine, sucralfate B. proton pump inhibitor, clarithromycin C. only proton pump inhibitor D. only H2 receptor antagonist E. only antacids

B. proton pump inhibitor, clarithromycin

293. WHO principles of Treatment of Cancer Pain include: A. oral medication on demand 3-5 times a day and "analgesic ladder" w/ 3 steps B. regular medication and "analgesic ladder" w/ 3 steps C. "analgesic ladder" w/ 3 steps or using non-opioids only D. using non-opioids or opioids or adjuvants E. the recommendation not to exceed maximum doses of opioids

B. regular medication and "analgesic ladder" w/ 3 steps

392. Which antiarrhythmic agent most likely can cause torsades de pointes? A. digoxin B. sotalol C. esmolol D. lidocaine E. verapamil

B. sotalol

160. 45-year old man had an episode of monomorphic ventricular tachycardia associated with ischemic heart disease. Which of the following drugs is the best choice in the treatment of rhythm disorders for this patient? A. verapamil B. sotalol C. digoxin D. adenosine E. quinidine

B. sotalol - non selective BB (VT)

169. A 45-year old obese patient is undergoing an abdominal surgery. Isoflurane was used for general anesthesia. The prolonged exposure to isoflurane can lead for this patient to: A. the shortening of time needed to wake up from general anesthesia B. the increased risk of post-surgery vomiting C. the shortening of the duration of post-surgery sedation D. the risk of hyperglycemia E. the risk of hyperkalemia

B. the increased risk of post-surgery vomiting

273. A patient suffers from allergic dermatitis. His physician has prescribed him a drug that is least likely to induce dry mouth. Which of the following it is?

Bilastine

93. Overdosing of salicylates can lead to all of the following EXCEPT: A. Increase in blood pH B. Nausea and vomiting C. Diabetes mellitus D. Marked hyperventilation E. Tinnitus (ringing or roaring in the ears)

C

593. Digoxin: 1. reduces heart rate 2. can cause heart blocks 3. is the treatment in atrial fibrillation in a patient with heart failure 4. is an inotropic positive agent 5. its therapeutic index is high A. 1,2,3 B. 1,2,3,4,5 C. 1,2,3,4 D. 5 E. 4,5

C. 1,2,3,4

592. Intravenous furosemide: 1. causes natriuresis 2. causes kaliuresis 3. has an indirect vasodilator effect 4. diuresis begins 10-20 minutes after an intravenous dose 5. high doses are ototoxic A. 1,2,3 B. 3,4,5 C. 1,2,3,4,5 D. 5 E. 1,5

C. 1,2,3,4,5

587. The management of coronary heart disease usually includes: 1. aspirin 2. glyceryl trinitrate (nitroglycerin) 3. prednisone 4. salbutamol 5. beta-adrenoceptor antagonist A. 1,2,3 B. 1,2,4 C. 1,2,5 D. 1,2 E. 1,2,3,4,5

C. 1,2,5

362. The recommended duration of treatment with antithyroid drugs is: A. 1 month B. 6 months C. 12-18 months D. 5 years E. 10 years

C. 12-18 months

345. Patients having a history of anaphylactic shock due to amoxicillin: 1. may be given a cephalosporin without concern 2. are at a high risk of anaphylactic shock when exposed to macrolide 3. are at high risk of immediate reaction to cephalosporin 4. are at a low risk of having a immediate reaction to monobactam A. 1,3,4 B. 2,3,4 C. 3,4 D. 4 E. 2,3

C. 3,4

222. How long before the planned dental surgery the antiplatelet effect of aspirin should be evaluated and (in case of need) the dose adjusted or discontinued? A. it depends on the patients's risk of bleeding and may be 12 hours B. to 2 days C. 7-10 days D. 1 day E. 12 hours F. 4 weeks

C. 7-10 days

398. The main difference between angiotensin receptor inhibitors and ACE inhibitors is related to: A. Inhibition of sympathetic nervous system B. Inhibition of aldosterone production and release C. Absence of dry cough as adverse effect D. Absence of hypotonia after first drop E. More effective protection against heart remodeling

C. Absence of dry cough as adverse effect

53. The main difference between angiotensin receptor inhibitors and ACE inhibition is based on: A. Inhibition of sympathetic nervous system B. Inhibition of aldosterone production and release C. Absence of dry cough as adverse effect D. Absence of hypotonic after the first dose E. More effective protection against heart remodelling

C. Absence of dry cough as adverse effect

452. A 45-year old hypertensive patient, felt worse when was in the dentistry office. His tongue suddenly became extremely swollen within few minutes. These symptoms could be the adverse effects of which of the following agents: A. Cyclosporine B. Lacidipine C. Acetylsalicylic acid D. Nitroglycerin E. Valsartan

C. Acetylsalicylic acid

6. Which among chlorhexidine antibacterial activities is the most closely related to its prevention of caries lesions formations in fissures and on interproximal tooth surfaces? A. Activity against E. coli B. Activity against Pseudomonas aeruginosa C. Activity against Streptococcus mutans D. Activity against Streptococcus aureus E. Activity against Streptococcus pyogenes

C. Activity against Streptococcus mutans

325. For a quick relief from bronchial obstruction in asthma one of the following agents should be used: A. Fluticasone propionate B. Salmeterol C. Albuterol D. Ciclesonide E. Omalizumab

C. Albuterol

202. The 1-line agents for therapy of infection caused by common intraoral pathogens are: A. Tetracyclines B. Metronidazole and its derivatives C. Aminopenicillins D. Macrolides E. Clindamycin and other lincosamides

C. Aminopenicillins

90. Patient F.R. requires hydrocortisone intravenously. This patient most probably suffers from: A. Rhinitis B. Megaloblastic anemia C. Anaphylactic shock D. Congestive heart failure E. Myocardial infarction

C. Anaphylactic shock

37. The use of chloramphenicol may result in: A. Bone marrow stimulation B. Phototoxicity C. Aplastic anemia D. Staining of teeth E. Alopecia

C. Aplastic anemia

458. Drug which decreases LDL-cholesterol serum levels and decreases triglyceride serum concentration, is: A. Probucol B. Fenofibrate C. Atorvastatin D. Cholestyramine E. Ezetimibe

C. Atorvastatin

103. A 75-year-old-male develops productive cough with blood-tinged sputum. He has high fever and Gram-positive cocci in clusters are found in the sputum smear. Chest x-ray shows increased density in the right upper lobe. Of the following antibiotics, which one is most likely to be ineffective? A. Telithromycin B. Amoxicillin C. Aztreonam D. Clarithromycin E. Clindamycin

C. Aztreonam

475. Which of the following can be used as anti-anxiety agents: A. Beta 2-mimetics B. Thiazides C. Benzodiazepines D. Dihydropyridines E. Incretins

C. Benzodiazepines

510. Antimaniac activity of carbamazepine is based on: A. The inhibition of 5-HT reuptake B. Increasing of GABA level C. Blocking of sodium channels D. The inhibition of NE reuptake E. The inhibition of NE release

C. Blocking of sodium channels

472. Etanercept: A. Blocks IL-2 B. Is an anti-platelet agent C. Blocks tumor necrosis factor alpha (TNF-a) D. Blocks cell-mediated immunity E. Is an anticancer antibody

C. Blocks tumor necrosis factor alpha (TNF-a)

384. Side effects like persistent cough, angioedema (swelling of lips, mouth, nose), hyperkalemia, is characteristic for: A. Losartan, candesartan, valsartan and telmisartan (X) B. Nifedipine, amlodipine C. Captopril, enalapril, ramipril, fosinopril D. Verapamil, diltiazem E. Hydroxyzine, cetirizine

C. Captopril, enalapril, ramipril, fosinopril

300. Extracerebral production of dopamine can be prevented by the inhibitor of dopa-decarboxylase that does not cross the blood-brain barrier, leaving intracerebral decarboxylation unaffected. This inhibitor is: A. Selegiline B. Entacapone C. Carbidopa D. Benserazide E. Tolcapone

C. Carbidopa

420. A woman has developed a moderately severe contact dermatitis reaction to a cosmetic preparation on her face and eyes. Which of the following agents administered topically would be most suitable for treating this condition? A. Terbinafine B. Griseofulvin C. Clobetasol D. Lidocaine E. Azelaic Acid

C. Clobetasol

228. Which of the following compounds is most likely to block ADP receptors and prevent platelet aggregation? A. Abciximab B. Aspirin C. Clopidogrel D. Prostacyclin E. Montelukast

C. Clopidogrel

106. A 23-year old male patient with glucose-6-phosphate deficiency induced anemia developed pneumonia. Which of the following agents should not be given to this patient? A. Aminoglycoside B. Vancomycin C. Co-trimoxazole D. Metronidazole E. Macrolide

C. Co-trimoxazole

580. Which from agents mentioned below are ... to surface application? A. Lidocaine B. Mepivacaine C. Cocaine D. Bupivacaine

C. Cocaine

548. Which of the following may be responsible for increased anticoagulant activity of enoxaparin? A. Decreased GI motility B. Deficiency of antithrombin C. Concurrent use of aspirin D. Concurrent use of vitamin K E. All of the above

C. Concurrent use of aspirin

469. Which one of the following poisons is improperly matched with the indicated treatment or antidote: A. Arsenic poisoning : dimercaprol (BAL) B. Iron poisoning : deferoxamine C. Cyanide poisoning : calcium gluconate D. Lead poisoning : calcium disodium edetate (EDTA) E. Copper poisoning : penicillamine

C. Cyanide poisoning : calcium gluconate

61. The pharmacological effects of biguanides include: A. Lipogenesis and stimulation of insulin secretion B. Decreased glucose absorption from gastrointestinal tract and stimulation of insulin secretion C. Decreased glucose absorption from gastrointestinal tract and increased insulin sensitivity in peripheral tissues D. Stimulation of insulin secretion from beta cells and decrease in lipoprotein concentration E. Lipogenesis and increased insulin sensitivity in peripheral tissues.

C. Decreased glucose absorption from gastrointestinal tract and increased insulin sensitivity in peripheral tissues

57. Potassium-sparing agents act in nephron on: A. Loop of Henle and distal convoluted tubule B. Proximal and distal convoluted tubules C. Distal convoluted tubule and collecting duct system D. Collecting duct and glomerulus E. Proximal convoluted tubule

C. Distal convoluted tubule and collecting duct system

111. A patient C.G. with chronic renal failure and comorbid anuria developed atypical pneumonia. Which of the following drugs would you recommend where dosing schedule need not be adjusted because of renal disease? A. Penicillin G B. Oxytetracycline C. Doxycycline D. Imipenem E. None of the above

C. Doxycycline

536. Choose the drug that doesn't match the others: A. Diazoxide B. Hydralazine C. Drotaverine D. Nifedipine

C. Drotaverine

104. Which of the following regimens should be used to rapidly alleviate biliary colic symptoms? A. Drotaverine im + metoclopramide im B. Atropine im + metoclopramide im C. Drotaverine im + hyoscine im D. Atropine im + hyoscine im E. Any of the above

C. Drotaverine im + hyoscine im

361. All of the following are factors affecting gastrointestinal absorption except: A. Area of absorbing surface B. Concentration of dissolved drug C. Drug binding to plasma proteins D. Drug-drug interactions in the gastrointestinal tract E. The pH of the gastrointestinal content

C. Drug binding to plasma proteins

97. Which of the following statements concerning morphine is incorrect: A. Oral morphine is the opioid of the 3rd step of "analgesic ladder" recommended by WHO B. Constipation is the most common adverse effect in patients treated with regular doses of morphine C. During prolonged treatment tolerance to miotic effect of morphine develops D. Morphine is a drug of choice in patient with excreted mainly via the biliary system E. Anaphylactic shock is a contraindication for morphine

C. During prolonged treatment tolerance to miotic effect of morphine develops

515. The correct statements regarding extrapyramidal side effects of antipsychotics is: A. Cholinomimetic agents are useful in the management of neuroleptic-induced Parkinsonism B. Acute dystonia reactions occur most commonly with low potency agents C. Excess dopaminergic activity is hypothesized to be a mechanism of tardive dyskinesia D. Tardive dyskinesia occurs early on in therapy and is reversible upon discontinuation of therapy E. Akathisia is characterized by oral facial dyskinesia

C. Excess dopaminergic activity is hypothesized to be a mechanism of tardive dyskinesia

393. A 55-year old man has benign prostatic hyperplasia and male pattern baldness. Which of the following agents would be most likely to alleviate both conditions? A. Cortisone B. Estradiol C. Finasteride D. Gonadotropins E. Testosterone

C. Finasteride

112. A 26 year old female with acquired immunodeficiency syndrome (AIDS) has developed cryptococcal meningitis. She refuses all intravenous medication. Which antifungal agent can be given orally to treat the meningeal infection? A. Ketoconazole B. Amphotericin B C. Fluconazole D. Nystatin E. Gentamycin

C. Fluconazole

55. White flecks on teeth with recommended doses and (rarely) yellowish-brown discoloration if recommended doses are exceeded can be produced by: A. Erythromycin B. Acyclovir C. Fluorides D. Calcium salts E. Metoclopramide

C. Fluorides

408. A man treated for Pneumocystis jiroveci infection subsequently develops hemolytic anemia. Which condition would predispose the patient to this side effect? A. Immunodeficiency B. Folate deficiency C. Glucose-6-phosphate dehydrogenase deficiency D. Iron deficiency E. Thiamine deficiency

C. Glucose-6-phosphate dehydrogenase deficiency

132. An 82-year old man with advanced prostatic carcinoma should undergo "chemical castration" with following drug: A. Gonadorelin B. Pegvisomant C. Goserelin D. Somatorelin E. Testosterone

C. Goserelin

232. The characteristic side effect of cyclosporine are all of the following except: A. Hirsutism B. Nephrotoxicity C. Haemorrhagic inflammation of the bladder D. CNS toxicity E. Gingival hyperplasia

C. Haemorrhagic inflammation of the bladder

354. Acetaminophen: A. Has similar effects on oxidative phosphorylation to aspirin B. Exhibits a high degree of protein binding in the blood C. Has good analgesic effect but poor anticoagulant or anti-inflammatory actions D. Is less potent centrally because it is inhibited by peroxidases E. All of above are true

C. Has good analgesic effect but poor anticoagulant or anti-inflammatory actions

32. Which sentence concerning the topical application of drugs to mucous membranes is true: A. This route of administration is never associated with systemic drug-dependent effects. B. Hydrophilic substances better penetrate across membranes than lipophilic C. In dentistry glucocorticosteroids can be applicated to mucous membrane D. Drugs cannot be administered on to the nasal mucosa E. Mucosal administration of drugs is contraindicated before dental extraction

C. In dentistry glucocorticosteroids can be applicated to mucous membrane

441. A 63-year old man recently diagnosed with Parkinson disease started treatment with levodopa/carbidopa. Which of the following actions most likely mediated the therapeutic effect of levodopa in the patient's disease? A. Inhibition of catechol-O-methyltransferase in the substance nigra B. Inhibition of dopa decarboxylase in the striatum C. Increased synthesis of dopamine in the subthalamic nucleus D. Down regulation of dopaminergic receptors in the striatum E. Increased synthesis of dopamine in the striatum

C. Increased synthesis of dopamine in the subthalamic nucleus

89. Effects of glucocorticoids do not include: A. Inhibition of leukotriene synthesis B. Altered fat deposition C. Increased synthesis of skin protein D. Increased blood glucose E. Reduction of circulating lymphocytes

C. Increased synthesis of skin protein

50. A 20-year-old male with herpes simplex of the lips is treated with acyclovir. Mechanism of action of acyclovir? A. Cross-linking of DNA B. Strand breakage of DNA C. Inhibition of viral DNA synthesis D. Inhibition of nucleotide intercoversions E. Inhibition of a viral kinase (all of the above)

C. Inhibition of viral DNA synthesis

121. A 76-year old obese male suffering big toe pain inflammation symptoms after a substantial meal. His plasma uric acid level was elevated. These symptoms were alleviated after administration of a mitotic spindle poison agent. This drug exhibits all of the following effects, EXCEPT: A. It is a uricosuric agent B. It inhibits the migration of granulocytes C. It can cause constipation D. It can produce agranulocytosis E. It can produce nausea

C. It can cause constipation

471. Indicate an incorrect pair: A. Rapid onset of thyroid action - triiodothyronine B. Used for hypothyricosis treatment - thyroxine C. Its principal metabolite inhibits peroxidase in the thyroid - triiodothyronine D. Preparation for surgery on a toxic goiter - lugol's iodine E. Treatment of thyroid carcinoma - 131 I

C. Its principal metabolite inhibits peroxidase in the thyroid - triiodothyronine

486. Which of the following anesthetics would be most likely to produce hallucinations and nightmares? A. Nitrous oxide B. Halothane C. Ketamine D. Thiopental E. Methohexital

C. Ketamine

320. Which one of the following drugs has the lowest chance of causing harm to the fetus and would be the safest in your pregnant patient for treating constipation over a 3 month period during her last trimester? A. Cholestyramine B. Erythromycin C. Lactulose D. Atropine E. Magaldrate

C. Lactulose

58. A patient displaying symptoms at acromegaly..............and enlargement of hand and wrist bone, can benefit from treatment with which of the following agents: A. Oxygen B. Digoxin C. Lanreotide (or octreotide) D. Amphotericin E. Any of the above

C. Lanreotide (or octreotide)

490. Patient presentation which is suggestive of local anesthetic allergy: A. Constipation B. Diarrhea C. Laryngeal edema D. Bradycardia E. Increased blood pressure

C. Laryngeal edema

184. Which of the following fluoroquinolones have enhanced gram-positive activity (i.e. coverage vs Streptococcus pneumoniae)? A. Ciprofloxazin B. Gentamicin C. Levofloxazin D. Penicillin G E. Dicloxazillin

C. Levofloxazin

64. Which from the following drugs can be used to treat hypothyroidism? A. Propyiothiouracil B. Oral solution of potassium iodide C. Levothyroxine D. Prednisone E. All of the above

C. Levothyroxine

242. A 45-year-old woman was undergoing the dental surgery. Her medical history was significant for long-term, severe chronic obstructive pulmonary disease (COPD) due to heavy smoking. Local anesthesia with a peripheral nerve block was planned. Which of the following local anesthetics would be contraindicated in this patient? A. Bupivacaine B. Procaine C. Lidocaine D. Prilocaine E. Tetracaine

C. Lidocaine

86. A 66-year old female on digoxin treatment (serum digoxin = 3 ng/ml) developed ventricular tachycardia. Which of the following agents could be most effective in treatment of this complication? A. Atropine B. Enalapril C. Lidocaine D. Verapamil E. Quinidine

C. Lidocaine

238. A local anesthetic was injected around the abscess in preparation for surgery. Which of the following tissue properties most likely account for the slower onset of local anesthetic action in infected tissues? A. Higher extracellular K+ B. High levels of para-aminobenzoic acid C. Lower extracellular pH D. Low vessel density E. High levels of drug-metabolizing enzymes

C. Lower extracellular pH

87. Which of the following agents would be a drug of choice for an obese patient (DM type 2) with peripheral insulin resistance? His fasting glucose is 105 mg/dl and postprandial glucose is 200 mg/dl. A. Detemir B. Repaglinide C. Metformin D. Lispro E. Glipizide

C. Metformin

36. Which of the following anticancer agents acts by blocking biochemical reactions with folic acid as their substrate? A. Vinblastine B. Doxorubicin C. Methotrexate D. Atomoxetine E. Cisplatin

C. Methotrexate

559. Folic acid antagonist binding to catalytic site of dihydrofolate reductase is: A. Vinblastine B. Cisplatin C. Methotrexate D. Mercaptopurine E. Fludarabine

C. Methotrexate

498. B.M a 42-year old male with type 1 diabetes mellitus. After repeated incidences of abdominal pain with nausea and vomiting, gastroparesis was diagnosed in B.M. Which of the following antiemetic drugs, that also exert prokinetic activity, may be useful in this patient? A. Ondansetron B. Chlorpromazine C. Metoclopramide D. Promethazine E. Scopolamine

C. Metoclopramide

128. A 75-year old woman was hospitalized because of pneumonia and treated with an intravenous antibiotic. On the 3rd day of therapy, she developed severe diarrhea. Stool was positive for Clostridium difficile toxin. What is the best treatment for her? A. Cefaclor B. Penicillin V C. Metronidazole D. Doxycycline E. Erythromycin

C. Metronidazole

476. Which of the following is used for patients with asthma: A. Omeprazole B. Clonidine C. Montelukast D. Digoxin E. Amlodipine

C. Montelukast

276. An agonist of μ (mu) opioid receptors and agonist of κ (kappa) receptors A. Naloxone B. Tramadol C. Morphine D. Fentanyl E. Buprenorphine F. Loperamide G. Oxycodone H. Methadone

C. Morphine F. Loperamide G. Oxycodone

501. Choose false statement - Original and generic preparations: A. Must have the same active ingredient B. Must have the same dose of active ingredient C. Must be therapeutically equivalent D. Must have the same route of administration E. Must have similar PK profile (Cmax, tmax, AUC are within 80-125% range as compared with the original preparation)

C. Must be therapeutically equivalent

135. A 50-year old female was treated for chronic myelogenous leukemia (CML) with a folic acid antagonist. Which of the following shall be the most likely adverse effects of this chemotherapeutic agent? A. Hepatotoxicity B. Nephrotoxicity C. Myelosuppression D. Cardiotoxicity E. Neurotoxicity

C. Myelosuppression

550. NSAIDs are contraindicated in patients with gastric ulcers because: A. NSAIDs inhibit gastric acid production B. NSAIDs increase the risk of renal insufficiency C. NSAIDs inhibit synthesis of prostaglandins in gastric mucosa D. NSAIDs cause Reye's syndrome E. NSAIDs cause hypersensitivity reactions

C. NSAIDs inhibit synthesis of prostaglandins in gastric mucosa

167. A 53-year-old man was taken to the emergency department because of dizziness and chest discomfort that apparently had been ongoing for over 5 hours. His vital signs were blood pressure 165/100 mm Hg, heart rate 50 bpm, respirations 22/min. Physical examination revealed signs of severe pulmonary congestion, and an anterior acute myocardial infarction. An appropriate therapy was instituted that included an intravenous infusion of which of the following drugs? A. Epinephrine B. Metoprolol C. Nitroglycerin D. Verapamil E. Nifedipine

C. Nitroglycerin - for MI, pulm con, angina, HPN. Acute IV treatment

189. Morphine is: A. Chemical name B. Proprietary name C. Nonproprietary name D. Code name E. Trade name

C. Nonproprietary name

630. Morphine is: A. Chemical name B. Proprietary name C. Nonproprietary name D. Code name E. Trade name

C. Nonproprietary name

604. Which of the following agents would you recommended to the patient with rheumatoid arthritis treated with high doses of NSAIDs to reduce the risk of peptic ulcers? A. Magnesium hydroxide B. Aluminum hydroxide C. Omeprazole D. Amoxicillin E. Sucralfate

C. Omeprazole - PPI

210. Chronic administration of corticosteroids (iatrogenic Cushing's syndrome) results in: A. Orthostatic hypotonia B. Dehydration C. Osteoporosis D. Hyperkalemia E. Hypercalcemia

C. Osteoporosis

124. Which of the following is NOT a phase II drug metabolising reaction? A. Acetylation B. Conjugation with glycine C. Oxidation D. Formation of mercapturic acid derivatives E. Glucuronidation

C. Oxidation - phase I

474. Proton pump inhibitors (PPI): A. PPIs inhibit primarily histamine-related HCl secretion B. PPIs inhibit gastrin release from G-cells C. PPIs inhibit nearly total HCl secretion D. PPIs inhibit both gastrin and HCl secretion E. PPIs inhibit primarily atropine-related HCl secretion

C. PPIs inhibit nearly total HCl secretion

355. A 65-year old hypertensive patient felt worse when was in dentistry office. On examination his BP was 90/60 mm Hg. Which of the following agents, could be the most likely reason of such BP decrease? A. Streptomycin B. Atropine C. Perindopril D. Norepinephrine E. Metformin

C. Perindopril

318. R.F. is a 45-year-old female with asthma who presents to her Physician's office with a 2-day history of fever, arthralgia, myalgias, and headache. She reports having a dry cough and a stuffy nose. A rapid influenza test is performed and is positive for Influenza B. Which of the following would be most appropriate for this patient at this time? A. Place patient on oseltamivir B. Place patient on didanosine C. Place patient on amantadine D. Place patient on interferon gamma E. Place patient on ribavirin

C. Place patient on amantadine

283. Which glucocorticoid is most likely to cause sodium and water retention? A. Desmopressin B. Betamethasone C. Prednisone D. Celecoxib E. Dexamethasone

C. Prednisone

305. R.L, a 37 y.o. man weighing 100 kg man has been experiencing retrosternal burning pain for approximately 4-6 weeks. The pain usually occurs after dinner while he is lying on the couch. The gastroesophageal reflux disease (GERD) was diagnosed in this patient. What is the most appropriate therapy of R.L.'s GERD? A. Antacids as a main therapy B. Misoprostol C. Proton pump inhibitor D. Sucralfate E. Bismuth colloid compounds

C. Proton pump inhibitor

126. Calcipotriene (synthetic 1,25-dihydroxyvitamine D3) is used for the topical treatment of: A. Acne B. Urticaria C. Psoriasis

C. Psoriasis

448. The following is an adverse effect of oral contraceptives: A. Bleeding from gastrointestinal tract B. Ventricular tachycardia C. Pulmonary thromboembolism D. Hypoglycemia E. Hypercalcemia

C. Pulmonary thromboembolism

42. With continued use of strong opioid analgesics tolerance develops to all of the following effects EXCEPT: A. Sedation B. Analgesia C. Pupillary constriction D. Euphoria E. All of the above

C. Pupillary constriction

473. Which of the following inhibits HCl secretion? A. Colloidal bismuth compound B. Sucralfate C. Ranitadine D. Digoxin E. Amoxicillin

C. Ranitadine

459. The rapid heart rate sometimes seen after nitroglycerin administration is best explained by: A. A direct positive chemotropic effect on the myocardium B. The ability of nitroglycerin in release norepinephrine from sympathetic nerve endings C. Reflex sympathetic discharge due to a fall in systemic blood pressure D. A decrease in intracranial pressure E. A toxic impairment of the heart muscle

C. Reflex sympathetic discharge due to a fall in systemic blood pressure

509. Systems on which antipsychotic agents exhibit pharmacologic action include all of the following EXCEPT: A. Cardiovascular system B. Autonomic nervous system C. Respiratory system D. Central nervous system E. Endocrine system

C. Respiratory system

479. Selegiline: A. Selectively inhibits dopamine receptors B. Selectively inhibits epinephrine reuptake C. Selectively inhibits MAO-B D. Selectively inhibits serotonin re-uptake E. Selectively inhibits GABA receptors

C. Selectively inhibits MAO-B

328. The most effective antiemetics, recommended as first-line agents in prophylaxis of anticancer chemotherapy-induced nausea and vomiting are: A. Dopamine antagonists B. Antihistaminic agents C. Serotonin antagonists D. Anticholinergic agents E. Cholinolytics

C. Serotonin antagonists

116. A 7-year old boy with asthma experienced severe dyspnea and cough. The emergency treatment he was given must have included: A. Budesonide in inhalation B. Montelukast C. Short acting beta-adrenergic agent in inhalation D. Theophylline sustained release E. Long acting beta-adrenergic agent in inhalation

C. Short acting beta-adrenergic agent in inhalation

172. A 47-year-old Hispanic man recently diagnosed with pulmonary tuberculosis started a multidrug treatment. One of the drugs he was given is active against most mycobacteria, many gram-positive and -negative bacteria, chlamydiae, rickettsiae and poxviruses. Which of the following drugs has such a broad antibacterial spectrum? A. Isoniazid B. Rifampin C. Streptomycin D. Ethambutol E. Pyrazinamide

C. Streptomycin

333. A 24-year old woman is treated with Praziquantel. What is the diagnosis? A. Giardiasis B. Amebiasis C. Taeniasis D. Malaria E. Trichomoniasis

C. Taeniasis

9. Which from agents mentioned below are limited to surface application? A. Lidocaine B. Mepivacaine C. Tetracaine D. Bupivacaine E. Articaine

C. Tetracaine

241. A 43-year-old man underwent surgery. Sevoflurane was used for general anesthesia. The drug has a minimum alveolar concentration (MAC) of 2%. Which of the following best describes the MAC of an inhalational anesthetic? A. The concentration of anesthetic needed for short surgery B. The maximal efficacy of the anesthetic C. The ED50 on a conventional quantal dose-response curve D. The blood/gas partition coefficient of the anesthetic E. The concentration of anesthetic in the inspired air

C. The ED50 on a conventional quantal dose-response curve

35. Which from the following statements about the pharmacokinetics of drugs in pregnancy is not correct: A. The total level of plasma protein declines B. The total amount of body fluids increases C. The total volume of distribution of drugs remains unchanged D. All of the above are true E. None of the above is true

C. The total volume of distribution of drugs remains unchanged

196. The therapeutic uses of isoxazolyl penicillins include: A. The treatment of every purulent infection B. The treatment of anaerobic infection C. The treatment of infections that are documented to be caused by staphylococci D. The treatment of infections that are documented to be caused by streptococci E. The treatment of Herpes simplex infection

C. The treatment of infections that are documented to be caused by staphylococci

296. Following oral administration, a drug is absorbed into the body, where it can exert its action. For a drug given orally, the primary site of drug absorption is: A. The esophagus B. The stomach C. The upper portion of the small intestine D. The upper portion of large intestine E. The annus

C. The upper portion of the small intestine

334. Which of the following is true about chronic use of opiate drugs? A. There is a very high degree of abuse liability when prescribing high doses of opiates to control pain in terminal patients B. Less lipid soluble opiates have higher abuse liability than more lipid soluble drugs C. There is more tolerance to the analgesia and euphoria caused by repeated use of opiates than to miosis and constipation D. Tolerance to opiates involves a decrease in the number of μ receptors in the membranes as well as a decrease in overall adenylate cyclase activity E. Their influence on CNS is limited

C. There is more tolerance to the analgesia and euphoria caused by repeated use of opiates than to miosis and constipation

294. In cancer pain, morphine is used, so what is the following statement is correct? A. Treating Mild pain according to 3 ladder of WHO B. Treat Moderate pain according to 3 step ladder of WHO C. Treat Severe pain according to 3 ladder of WHO

C. Treat Severe pain according to 3 ladder of WHO

556. Atypical facial pain and pain associated with temporomandibular joint dysfunction syndrome is termed as neuropathic pain. That is why the treatment should especially consider: A. NSAIDs B. Morphine C. Tricyclic antidepressants D. All of the above are true E. None of the above is true

C. Tricyclic antidepressants

445. Drugs excreted by the kidney cannot be eliminated from the body as the result of: A. Glomerular filtration B. Tubular secretion C. Tubular excretion D. Active transport E. Passive transport

C. Tubular excretion

535. A drug is currently first-choice drug in the management of absence seizures as well as partial, primary generalized and tonic-clonic seizures is: A. Topiramate B. Ethosuximide C. Valproic acid D. Carbamazepine E. Pregabalin

C. Valproic acid

31. Which of the clinical situations mentioned below mandatory contraindication for the oral administration of any drug? A. Respiratory insufficiency B. Cardiac insufficiency C. Vomiting D. Constipation E. Mental disorder

C. Vomiting

120. T.S. a male with paroxysmal atrial fibrillation has been on drug A to prevent thromboembolism. This drug, however, produced severe bleeding which required fresh frozen plasma and vitamin K for treatment. Drug A is which one of the following? A. Heparin B. Ticlopidine C. Warfarin D. Aspirin E. Streptokinase

C. Warfarin (acenocoumarol)

176. A 40-year-old patient is planned to a minor surgery: the extraction of ingrown toenail. The physician ordered an anxiolytic drug (X) and a general anaesthetic with the antiemetic activity (Y). Choose the most likely drugs X and Y combination: A. X: isoflurane; Y: etomidate B. X: etomidate; Y: fentanyl C. X: midazolam; Y: propofol D. X: desflurane; Y: morphine E. X: ketamine; Y: atracurium

C. X: midazolam; Y: propofol - benzo/GA

390. Which of the following actions most likely mediates the therapeutic effect of hydralazine? A. reduction of angiotensin II secretion B. increased preload C. afterload reduction D. inhibition of sympathetic activity E. decreased cardiac contractility

C. afterload reduction (by directly dilating the arteries)

544. Of the many types of adrenergic receptors found throughout the body, which one is most likely responsible for the cardiac stimulation observed following IV injection of epinephrine? A. b-2 adrenergic receptors B. a-2 adrenergic receptors C. b-1 adrenergic receptors D. b-3 adrenergic receptors E. a-1 adrenergic receptors

C. b-1 adrenergic receptors

608. 30-year-old pregnant women (1st trimester) requires antibiotic for orodental infection due to mixed Gram-positive typical bacteria. The agent of choice for her is: A. penicillin G B. metronidazole C. erythromycin D. amoxicillin E. ciprofloxacin

C. erythromycin

144. Which of the following drugs increases only glucose-dependant insulin secretion, decreases apoptosis and stimulates differentiation of pancreatic beta cells, delays gastric emptying, decreases appetite, reduces body weight and waist circumference in patients with type 2 diabetes: A. glimepiride require B cells B. metformin C. exenatide D. sitagliptin E. acarbose

C. exenatide - GLP1 analog (inc. insulin, dec. glucagon, dec. gastric emptying)

177. A patient is treated by the urologist because of benign prostatic hypertrophy. Prescribed drugs block the conversion of testosterone to its more active intracellular form-dihydrotestosterone, The most likely it is: A. flutamide B. cyproterone C. finasteride D. danazole E. goserelin

C. finasteride - 5a reductase inhibitor Finasteride = USES: BPH, baldness MOA 5-alpha reductase inhibitor, preventing conversion of testosterone to dihydrotestosterone (DHT). DHT is responsible for hair loss and prostate enlargement. Caution teratogenicity. Other common drugs in BPH are alpha-1-antagonists such as Prazosin and Tamsulosin

25. Which of the following factors describes the main difference between first- and second-generation antihistamines? A. first generation agents have less CNS effects. B. second generation agents have more side effects. C. first generation agents are more sedating D. second generation agents are less selective E. second generation agents are only used for nausea prevention

C. first generation agents are more sedating

385. Which of the following clinical situation may be an indication for the use of ARBs (angiotensin II receptor blocker/antagonist), but not in ACE-1? A. heart failure in pregnant women B. heart failure in patients with bilateral renal arteries stenosis C. heart failure in patients with angioedema D. heart failure in patients with end-stage renal failure E. acute heart failure associated with cardiac shock

C. heart failure in patients with angioedema

623. Which of the following is true for blood-brain barrier: A. ionized drugs distribute very well to the CNS B. polar drugs distribute very well to the CNS C. inflammation may increase the ability of ionized drugs to cross blood-brain barrier D. A and B are true E. A, B, C are true

C. inflammation may increase the ability of ionized drugs to cross blood-brain barrier

590. Aspirin: A. needs to be given twice daily to prevent platelet cyclo-oxygenase resynthesis within the dose interval B. predisposes to peptic ulceration C. irreversibly inhibits cyclooxygenase D. has no effect on bleeding time E. A and B are true

C. irreversibly inhibits cyclooxygenase - yes, inhibit COX-1 and 2; selective COX-2, irreversibly covalently acetylates active site of the enzyme → permanent action

289. Pt has been taking high doses of NSAIDs for osteoarthritis. Which prostaglandin agent is approved for prevention of gastric ulceration caused by NSAIDs? A. ranitidine B. telenzepine C. misoprostol D. alprostadil E. sucralfate

C. misoprostol

401. A woman who has developed resistance for cyclophosphamide and doxorubicin is given a monoclonal antibody that releases a cytotoxic isotope after binding to CD20 receptors. She is most likely being treated for which cancer? A. breast cancer B. ovarian cancer C. non-Hodgkin lymphoma D. multiple myeloma E. melanoma

C. non-Hodgkin lymphoma

15. Effect(s) associated with insulin-induced hypoglycemia: A. rigidity B. decreased heart rate C. sweating D. all of the above E. A and B

C. sweating

161. 52-year old woman is suffering from essential hypertension for many years. Which of the combinations of hypotensive drugs must be avoided? A. metoprolol + indapamide B. amlodipine + methyldopa C. valsartan + perindopril D. amlodipine + hydrochlorothiazide E. indapamide + amlodipine

C. valsartan + perindopril (ARB+ACEi)

264. Which of the following is a potent inhibitor of cytochrome?

Clarithromycin

227. A 53-year-old man presented to the symptoms of GI mucosa candidiasis for the past week. The man had a 10-year history of poorly controlled diabetes mellitus. A diagnosis was made, and an oral therapy was prescribed. Which of the drugs would be appropriate for this patient? A. Nystatin B. Griseofulvin C. Streptogramine D. Fluconazole E. Terbinafine

D

272. Which of the following diuretics is most likely to induce dry mouth symptoms in your patient? A. indapamide B. hydrochlorothiazide C. spironolactone D. furosemide E. amyloride

D

313. When the combined action of two drugs is greater than the sum of their individual actions, it is known as: A. Indifference B. Idiosyncrasy C. Cumulative action D. Synergy E. Additive

D

38. Selective beta 2-adrenomimetic causes: A. Vasoconstriction B. Vasodilation of skeletal muscle blood vessels C. Bronchoconstriction D. Bronchodilation E. B and D are true

D

98. Mr Kowalski is put on intravenous heparin therapy for thrombophlebitis (with a loading dose of 10.000 U followed by continuous infusion of 1500 IU/hour). The APTT after six hours was 20 sec (normal range is less than 60 sec). How should heparin dosage in Mr Kowalski be changed? A. Dosage of heparin should be increased B. Dosage of heparin should be decreased C. Heparin should be immediately discontinued D. Dosage of heparin is appropriate E. Heparin should be immediately discontinued and warfarin should then be initiate

D

277. Which chlorhexidine preparation is recommended for daily plaque control? A. 0.001%~0.002% mouthwash B. 0.01%~0.02% mouthwash C. 1.0%~2.0% mouthwash D. 0.1%~0.2% mouthwash E. 10%~20% mouthwash

D. 0.1%~0.2% mouthwash

561. Warfarin: 1. prevents the hepatic synthesis of the vitamin K-dependent coagulation factors II, VII, IX and X 2. is structurally closely related to vitamin K 3. should initially be given as a subcutaneous loading dose 4. during life-threatening bleeding can be reversed by vitamin K and fresh frozen plasma 5. anticoagulant effect is monitored by measurement of the prothrombin time/international normalized ratio (INR) A. 1,2,3 B. 2,3,4 C. 4,5 D. 1,2,3,4,5 E. 3,4

D. 1,2,3,4,5

483. The best example of non-depolarizing muscle relaxant is: 1. Atracurium 2. ... (either Vecuronium or Cisatracurium) 3. Pancuronium 4. Succinylcholine 5. Rocuronium A. 1,4 B. 1,3,4 C. 4,3 D. 1,2,3,5 E. 1,2,5

D. 1,2,3,5

466. Features of the opioid withdrawal syndrome include: 1. Yawning 2. Dry mouth 3. Respiratory depression 4. Constipation 5. Tremor A. 1,2,3 B. 2,4,5 C. 2,4 D. 1,5 E. 1,2,3,4,5

D. 1,5

574. Nitroglycerin used as aerosol relieves chest pain in: A. 6-12 hours B. 30-40 min C. 1-2 hours D. 1-5 min E. 15-20 min

D. 1-5 min

343. K.S, a 54-year-old male is suffering from type I diabetes mellitus and chronic renal insufficiency. Which of the following vitamin D derivatives should be use in this patient? A. Cholecalciferol B. Ergocalciferol C. Calcifediol (25(OH)D3) D. 1α-hydroxyvitamin D3 E. Calcipotriene

D. 1α-hydroxyvitamin D3

281. Drug biotransformation, which opinion is true? A. mainly, but not exclusively in the liver B. Products are usual (but not always) less biologically active C. Biotransformation products are never toxic D. A & B E. A, B, & C

D. A & B

231. Adverse effects associated with cyclosporine include: A. is nephrotoxic B. causes gingival hyperplasia C. causes alopecia D. A and B are true E. B and C are true

D. A and B are true

600. Metoclopramide: A. is effective in vomiting B. is ineffective in nausea C. increases the rate of gastric emptying D. A and C are true E. A,B,C are true

D. A and C are true

540. For optimal anesthetic action, a drug should be: A. A weak acid in ionized form B. A weak acid in non-ionized form C. A weak base in non-ionized form D. A weak base in ionized form E. Partially in non-ionized and ionized form

D. A weak base in ionized form

437. The epinephrine induces prolonged duration of the lidocaine effect. It is mediated by the activation of which of the following adrenoceptors? A. B1 B. B3 C. B2 D. A1 E. A2

D. A1

310. Which one from the following antianginal agents decreases or even reverses heart remodeling: A. Nitrates like nitroglycerin B. Ca-channels antagonists like dihydropyridines C. Ca-channels antagonists like verapamil D. ACE inhibitors like enalapril E. Nitrates like isosorbide mononitrate

D. ACE inhibitors like enalapril

209. Rapid withdrawal after dexamethasone administration can cause: A. Arthralgia B. Malaise C. Fever D. Acute adrenal insufficiency E. Hypercalciuria

D. Acute adrenal insufficiency

376. Which of the following complications can be observed with a cyclosporine? A. Nephrotoxicity B. Hypertension C. Hirsutism D. All of the above are true E. None of the above are true

D. All of the above are true

463. The following are likely to be effective in the prevention from acute episodes of gout (for chronic treatment): A. Furosemide B. Acetylsalicylic acid C. Alendronate D. Allopurinol E. Diclophenac

D. Allopurinol

187. The primary role in empiric treatment of community acquired pneumonia has one of the following agents: A. Vancomycin B. Penicillin G C. Carbapenem D. Amoxicillin/Clavulanate E. ---penem

D. Amoxicillin/Clavulanate

13. Gancyclovir is: A. An anti influenza virus agent B. A nucleoside antiretroviral agent HIV infection C. An integrase inhibitor used for Herpes simplex type 1 infection D. An anti herpes virus agent used for cytomegalovirus infection E. A protease inhibitor used for hepatitis

D. An anti herpes virus agent used for cytomegalovirus infection

337. A 39-year old patient with rheumatoid arthritis is treated with the drug X, which is a recombinant form of an endogenous antagonist of the interleukin-1 receptor. The drug X is: A. Hydrocortisone B. Docetaxel C. 5-fluorouracil D. Anakinra E. Vinblastine

D. Anakinra

464. Which of the following is an agent recommended as the first choice for patients with hypercholesterolemia? A. Fenofibrate B. Ezetimibe C. Colestipol D. Atorvastatin E. Probucol

D. Atorvastatin

493. Which of the following statements is false? A. antiarrhythmic agents can induce severe arrhythmias B. nitroglycerin is an agent for chronic treatment C. beta-blockers are recommended for the treatment of acute anginal pain D. B and C are false E. all of above are false

D. B and C are false

46. Which from the following agents should be considered as responsible for xerostomia? A. Cholinomimetic agent B. Cholinolytic agent C. Tricyclic antidepressant D. B and C are true E. A and C are true

D. B and C are true

625. The ideal disinfectant: A. kills exclusively vegetative forms of bacteria B. is noncorrosive C. is non-toxic D. B and C are true E. A, B, C are true

D. B and C are true

489. The use of an antibiotic which antimicrobial activity is maintained within several hours although its concentration in plasma is below MIC (minimal inhibitory concentration) could be associated with: A. High risk of hypersensitivity B. Resistance development C. Once daily dose schedule of administration D. Bad tolerance with prolonged treatment E. Higher risk of kidney insufficiency

D. Bad tolerance with prolonged treatment

576. Antimanic activity of carbamazepine is based on: A. Inhibition of 5-HT response B. Inhibition of NE release C. Increasing of GABA level D. Blocking of sodium channels E. Inhibition of NE reuptake

D. Blocking of sodium channels

233. All statements about cyclosporine is true except: A. Binds to immunophilin called cyclophillin B. Causes gingival hyperplasia C. Suppreses macrophages activation D. Blocks IL-10 synthesis E. It is one from the calcineurine inhibitors

D. Blocks IL-10 synthesis - blocks expression of IL-2, inhibits release of IL-1

240. Which of the following may result from an overdose of insulin? A. Infection B. Allergy C. Hyperglycemia D. Blurred speech E. Diabetes Insipidus

D. Blurred speech

365. An 87-year old woman came to her doctor because of oral thrush. She has been taking the drug X by inhalation for two weeks because of her allergic asthma. She has been informed that she should wash her mouth after the drug X inhalation but she didn't do it. The drug X probably is: A. Salbutamol B. Theophylline C. Zafirlukast D. Budesonide E. Salmeterol

D. Budesonide

65. A 29-year-old male is allergic to benzocaine. Which of the following local anaesthetics could safely be used? A. Cocaine B. Norepinephrine C. Tetracaine D. Bupivacaine E. Procaine

D. Bupivacaine

284. Drugs bound to albumin: A. Effectively cross the blood brain barrier B. Often contain quaternary nitrogens C. Have a large volume of distribution D. Can under go competition with other drugs for albumin binding sites E. Are easily filtered by the glomerulus

D. Can under go competition with other drugs for albumin binding sites

575. At therapeutic doses, which of the following anticonvulsants causes least degree of sedation: A. Clonazepam B. Phenobarbital C. Phenytoin D. Carbamazepine E. Clorazepate

D. Carbamazepine

504. At therapeutic doses, which of the following anticonvulsants causes the least degree of sedation: A. Clorazepate B. Clonazepam C. Phenytoin D. Carbamazepine E. Phenobarbital

D. Carbamazepine - dizziness, nausea, aplastic anemia, agranulocytosis

70. A patient who is allergic to penicillin could be switched on the following antibiotics, EXCEPT: A. Macrolide B. Tetracycline C. Clindamycin D. Cephalosporin E. Aminoglycoside

D. Cephalosporin (another beta-lactam)

349. Hyoscine (scopolamine) is often administered intramuscularly on emergency basis in: A. Constipation B. Biliary colic C. Persistent dyspepsia and bloating D. Chemotherapy induced vomiting E. Peptic ulcer disease

D. Chemotherapy induced vomiting

557. Drug X is highly neurotoxic agent, which cross-links DNA strands. It is: A. G-mercaptopurine B. Vincristine C. Bevacizumab D. Cisplatin E. Daunorubicin

D. Cisplatin

461. In 45-year old man the localized infection confined to the apical tissues of nonvital teeth (periapical abscess) was diagnosed. Which antibiotic among mentioned below is the agent of choice except manual procedures (drainage)? A. Penicillin V B. Amphotericin C. Co-trimoxazole D. Clindamycin E. Vancomycin

D. Clindamycin

203. Which of the following agents causes sustained vasoconstriction? A. Procaine B. Bupivacaine C. Prilocaine D. Cocaine E. Lidocaine

D. Cocaine

298. Which one of the following agents causes vasoconstriction? A. Procaine B. Bupivacaine C. Prilocaine D. Cocaine E. Lidocaine

D. Cocaine

400. A man being treated for testicular cancer experiences ototoxicity and nephrotoxicity. The drug causing this adverse effect inhibits cancer growth by which mechanism? A. Folate antagonism B. Inhibition of tyrosine kinase C. Inhibition of epidermal growth factor receptors D. Cross-Linking of DNA E. Formation of free radicals

D. Cross-Linking of DNA

419. After receiving low dose of dexamethasone, a patient is found to have a plasma cortisol level of 20 mcg/dL the next morning (should be less than 5 mcg/dL). Which disorder is most likely to occur in this patient? A. Congenital adrenal hyperplasia B. Chronic adrenal insufficiency C. Conn disease D. Cushing disease E. Acute renal insufficiency

D. Cushing disease

633. Antimuscarinic drugs are used in dentistry to: A. Decrease bleeding B. Decrease patient's excitation and nervousness C. Increase effectiveness of local anesthetics D. Decrease salivation E. Decrease local anesthetics toxicity

D. Decrease salivation

562. A man with hyperlipidemia is treated with an agent that increases expression of lipoprotein lipase. Which of the following effects would be most likely result from this action? A. Increased LDL-C levels B. Decreased HDL-C levels C. Increased serum triglyceride levels D. Decreased serum triglyceride levels E. Decreased LDL-C levels

D. Decreased serum triglyceride levels

629. A 45-years-old patient was put on aminopenicillin (amoxicillin) given orally as prevention against endocarditis. Which factor from mentioned below could be the most likely responsible for ineffective treatment: A. Infection B. Anemia C. High carbohydrate diet D. Diarrhea E. Hyperlipoproteinemia

D. Diarrhea

438. Potassium channel opening in smooth muscle cells: A. Propranolol B. Hydralazine C. Verapamil D. Diazoxide E. Hydrochlorothiazide F. Furosemide G. Enalapril H. Clonidine I. Losartan J. Spironolactone K. Methyldopa L. Labetalol

D. Diazoxide

54. All of the following agents decrease blood pressure EXCEPT: A. Vasodilators B. Beta-blockers C. Selective alpha, adrenolytic D. Digoxin E. Calcium-channel antagonists

D. Digoxin

157. Which of the following antihistamines would be best used to treat mild nausea and vomiting caused by motion sickness? A. Cetirizine B. Ketotifen C. Loratadine D. Dimenhydrinate E. Roflumilast

D. Dimenhydrinate

324. A 62-years old man with akinesia, rigidity, tremor at rest, postural instability and gait disturbance (Parkinson) will not profit by dopamine treatment, because: A. This patient should be treated with antagonists of dopamine receptors B. This patient will profit by morphine only C. This patient's proper treatment is only clonazepam D. Dopamine itself cannot penetrate the blood-brain barrier E. Succinylcholine is of the best importance drug for this patient

D. Dopamine itself cannot penetrate the blood-brain barrier

100. Female patient L.K. received treatment for hypertension related chronic renal failure (serum Na=133mmol/; serum K=6 mmol/l). Which of the following agents? A. Furosemide B. Losartan C. Spironolactone D. Enalapril E. Thiazide

D. Enalapril

139. 44-year old patient is treated with a certain drug because of anemia. During the therapy nausea and upper gastric discomfort develops. This patient was probably treated with: A. Deferoxamine B. Iron-dextran C. Folic acid D. Ferrous sulfate E. Vitamin B12

D. Ferrous sulfate

208. Which of the following diuretics is at the highest risk of hypokalemia-induced arrhythmia? A. Hydrochlorothiazide B. Spironolactone C. Acetazolamide D. Furosemide E. Amiloride

D. Furosemide

374. Mrs H is a 53-year old female with 22 yr h/o of hypertension. She presents to your clinic with a prescription for a diuretic. Recently she was c/o increasing tiredness, fatigue, some confusion and nighttime urination. It is noted that her weight increased 5 kg and her creatinine concentration has increased to 9 mg/dl. You notice that she has just been diagnosed with chronic renal failure this week. What is the best diuretic to begin on Mrs H.? A. Hydrochlorothiazide B. Metolazone C. Spironolactone D. Furosemide E. Mannitol

D. Furosemide

410. A man being treated for chlamydial urethritis presents with severe erythema over his upper body after exposure to sun. Which substance might reduce oral bioavailability of this antibiotic? A. Folic acid B. Ascorbic acid C. Vitamin B12 D. Gastric antacids E. Alcohol

D. Gastric antacids

511. Abarelix is a drug with one of the following mechanisms of action. Please point the correct mechanism of action? A. Androgen receptor antagonism B. GnRH agonism C. Progesteron receptor antagonism D. GnRH antagonism E. Estrogen receptor modulation

D. GnRH antagonism

131. Which of the following is an effect of vitamin D overload? A. Hypocalcemia and hypophosphatemia B. Hypercalcemia and hypophosphatemia C. Hypocalcemia and hyperphosphatemia D. Hypercalcemia and hyperphosphatemia E. None of the above

D. Hypercalcemia and hyperphosphatemia

371. Typical effect of marijuana use produces: A. Bradycardia B. Dysphoria C. Hypertension D. Impairment of short-term memory E. Sensation that time is passing faster

D. Impairment of short-term memory

538. All of the following can be blocked by atropine pretreatment except: A. Salivation induced by neostigmine B. Vagal bradycardia C. Miosis induced by infusion of acetylcholine D. Increased blood pressure induced by nicotine poisoning E. Sweating induced by injection of pilocarpine

D. Increased blood pressure induced by nicotine poisoning (b/c atropine acts on muscarinic receptors)

373. The most accepted mechanism of action of a local anaesthetic is: A. Alteration of the resting membrane potential B. Inhibition of potassium influx C. Prevention of sodium efflux D. Inhibition of sodium influx E. Decrease in the membrane potential

D. Inhibition of sodium influx

77. The most accepted mechanism of action of local anesthetic is: A. Alteration of the resting membrane potential B. Inhibition of potassium influx C. Prevention of sodium efflux D. Inhibition of sodium influx E. Decrease in the membrane potential

D. Inhibition of sodium influx

94. Patient W.U. with severe heart failure felt worse last week - he suffered from severe dyspnoea and his edema exacerbated. His physician decided to give him the most potent of available diuretics. This diuretic is/belongs to: A. Inhibitors of Na - Cl symport B. Inhibitors of carbonic anhydrase C. Inhibitors of renal epithelial sodium channels D. Inhibitors of Na-K-Cl symport E. Antagonists of mineralocorticoid receptor

D. Inhibitors of Na-K-Cl symport

205. A 5-year-old child become ill while visiting relatives who have a farm in Arkansas. His symptoms include severe abdominal cramps with vomiting and diarrhea and profuse lacrimation and salivation. Pupillary constriction is marked. The most likely cause is exposure to: A. Antifreeze B. Lead-based paint C. Rat poison D. Insecticides E. Herbicides

D. Insecticides

584. Digoxin: A. Is recommended as a first line therapy of diastolic HF B. ... the progression of HF C. Should be added to an ACE-I before a beta-blocker in symptomatic HF D. Is indicated for the therapy of systolic, symptomatic HF E. Is recommended as a very potent diuretic agent

D. Is indicated for the therapy of systolic, symptomatic HF

416. A woman is placed on a drug that increases insulin sensitivity throughout AMP-kinase activation. Which adverse effect commonly results from taking this medication? A. Increased risk of heart failure B. Increased triglyceride and LDL cholesterol levels C. Diarrhea D. Lactic acidosis E. Hypoglycemia

D. Lactic acidosis

117. A 19-year pregnant woman requires treatment with an antihypertensive agent because of EPH gestation. All of the following drugs can be used in this pregnant women with the exception of: A. Verapamil B. Methyldopa C. Metoprolol D. Losartan E. Hydrochlorothiazide

D. Losartan

141. Which of the following drug combinations is associated with increased risk of severe bradycardia? A. Amlodipine + Furosemide B. Lercanidipine + Digoxin C. Digoxin + Furosemide D. Metoprolol + Digoxin E. Metoprolol + Indapamide

D. Metoprolol + Digoxin - beta blocker/ Na K ATPase

80. A 45-year old female has been found to have cholangitis and persistent diarrhea. Her physician suspects parasitic infection and orders duodenal contents examination and immunoenzymatic tests. After confirming his initial diagnosis the physician should prescribe: A. Chloroquine B. Piperazine C. Doxycycline D. Metronidazole E. Amoxicillin

D. Metronidazole

351. Minimum alveolar concentration (MAC) of inhaled anesthetics means: A. Minimum oxygen concentration compatible with life during anesthesia B. Minimum concentration of nitrous oxide that induces stage 2 of anesthesia C. Minimum alveolar concentration of anesthetic at 1 atmosphere that produces immobility in 100 percent of patients exposed to noxious stimulus D. Minimum alveolar concentration of anesthetic at 1 atmosphere that produces immobility in 50 percent of patients exposed to noxious stimulus E. Alveolar concentration of an anesthetic during recovery

D. Minimum alveolar concentration of anesthetic at 1 atmosphere that produces immobility in 50 percent of patients exposed to noxious stimulus

533. Which of the following drugs has a mechanism of action connected with the inhibition of inosine monophosphate dehydrogenase? A. Cyclophosphamide B. Tacrolimus C. Basiliximab D. Mycophenolate mofetil E. Cyclosporine

D. Mycophenolate mofetil

523. The most common adverse effect(s) of cisplatin is which of the following? A. Cardiac toxicity B. Hepatotoxicity C. Alopecia D. Nausea and vomiting E. All of the above

D. Nausea and vomiting

101. The most characteristic difference between verapamil and nitrendipine considering their pharmacological effects is that: A. Verapamil can induce ventricular fibrillation, while nitrendipine can't B. Verapamil can induce peripheral edema, while nitrendipine is not C. Nitrendipine can cause bradycardia, while verapamil is rather responsible for tachycardia D. Nitrendipine can induce peripheral edema, while verapamil is not E. Verapamil can cause bradycardia, while nitrendipine is rather responsible for tachycardia

D. Nitrendipine can induce peripheral edema, while verapamil is not

102. A 49-year-old patient with symptoms of malignant hypertension was admitted to intensive care unit and treated with IV infusion of a certain agents. This agent causes vasodilation of arteries along with dilation of veins. He was treated with: A. Minoxidil B. Furosemide C. Metoprolol D. Nitroprusside E. Hydralazine

D. Nitroprusside

356. An 80-year old man is increasingly forgetful and his wife is afraid he is developing Alzheimer's disease. You are considering prescribing an Anti-AChE drug to see if this will decrease his forgetfulness. Before making this prescription, you want to be sure that these drugs are suitable given the patient's medical history. Out of the possible preexisting conditions listed below, you should be least concerned about: A. Asthma B. Weak atrio-ventricular conduction C. Glaucoma D. Obstruction of the GI tract E. Sinus bradycardia

D. Obstruction of the GI tract

414. A woman with osteolytic bone cancer is treated with a drug that reduces the serum calcium level. Which drug is indicated for this purpose? A. Calcitonin B. Teriparatide C. Calcitriol D. Pamidronic acid E. Strontium ranelate

D. Pamidronic acid

522. Drugs used in treatment of generalized tonic-clonic seizures include all of the following except: A. Carbamazepine B. Diazepam C. Phenytoin D. Phenobarbital E. Ethosuximide

D. Phenobarbital

76. Which of the following may cause methemoglobinemia? A. Lidocaine B. Ropivacaine C. Bupivacaine D. Prilocaine E. Cocaine

D. Prilocaine

207. Plasma cholinesterase is an enzyme responsible for breakdown of: A. Articaine B. Bupivacaine C. Mupivacaine D. Procaine E. Pilocaine

D. Procaine - (Pseudocholinesterase)

394. Potential indications to testosterone administration include all of the following except for: A. Androgen replacement in hypogonadal B. Refractory anemia C. Prevention of osteoporosis D. Prostate cancer E. Treatment for low sperm count

D. Prostate cancer

122. Which of the following can be used in treatment of toxoplasmosis? A. Pyrantel pamoate B. Piperazine C. Mebendazole D. Pyrimethamine, sulfonamide E. Albendazole, pyrantel pamoate

D. Pyrimethamine, sulfonamide

412. A 52-year-old postmenopausal woman is placed on a drug that decreases osteoclast activation but may intensify menopausal hot flashes. Which drug was most likely given to this patient? A. Alendronate B. Strontium ranelate C. Calcitonin D. Raloxifene E. Teriparatide

D. Raloxifene

406. One of the drugs used to treat a man with leprosy or TB causes orange-red colored urine. This drug inhibits the synthesis of which cell component? A. Mycolic acid B. Glycoproteins C. Membrane lipids D. Ribonucleic acids E. Folic acid

D. Ribonucleic acids

425. Which agent listed below is an antipsychotic that can improve both positive and negative symptoms of schizophrenia? A. Chlorpromazine B. Haloperidol C. Fluoxetine D. Risperidone E. Sertraline

D. Risperidone

430. Patient with anaphylactic shock should be treated (among others) with: A. Salbutamol 2.5-5 mg IM B. Salbutamol 20.5-50 mg s.c C. Salbutamol 0.25-0.5 mg as inhalation D. Salbutamol 2.5-5 mg as nebulization E. Salbutamol 2.5-5 mg IV

D. Salbutamol 2.5-5 mg as nebulization

352. M.P, a 55-year old woman 6 months after her last menstrual bleeding is planned to start HRT (hormonal replacement therapy). Which of the following is a contraindication for HRT? A. Hypocalcemia B. Hypercalcemia C. Elevated plasma level of cholesterol D. Severe liver insufficiency E. All of the above are true

D. Severe liver insufficiency

632. Nonsteroidal anti-inflammatory drugs can cause all of the following EXCEPT: A. Peptic ulceration, renal insufficiency B. Allergy, bleeding C. Exacerbation of asthma, thrombocytopenia D. Tachycardia, heart remodeling E. Liver impairment, peripheral edema

D. Tachycardia, heart remodeling

44. Nonsteroidal anti-inflammatory drugs can cause all of the following EXCEPT: A. Peptic ulceration, renal insufficiency B. Allergy, bleeding C. Exacerbation of asthma, thrombocytopenia D. Tachycardia, heart remodelling E. Liver impairment, peripheral edema

D. Tachycardia, heart remodelling

278. A patient report he is allergic to some drug, which of the following drug is not suitable for him? A. Prilocaine B. Lidocaine C. Bupivacaine D. Tetracaine E. Mepivacaine

D. Tetracaine

506. Carbidopa is used: A. As a successful monotherapy at the first stage of disease B. To reduce side effects during the treatment of psychotic episodes C. To decrease the incidence of gastrointestinal side effects associated with levodopa D. To conjugate with levodopa to block peripheral conversion to dopamine E. To treat psychotic episodes

D. To conjugate with levodopa to block peripheral conversion to dopamine

407. A woman being treated for urinary drug infection complains of heel pain and is found to have inflamed Achilles tendon. Bacterial resistance to the agent causing this side effect may result from decreased binding to which cell component? A. Membrane phospholipid B. RNA polymerase C. Folate reductase D. Topoisomerase E. Divalent Cations

D. Topoisomerase

295. The pharmacotherapy of chronic high-grade cancer pain with neuropathic component (shooting pain, seizures) is based on the use of: A. Anticonvulsants and glucocorticosteroids B. Potent opioids and anticonvulsants C. Oxycodone and amitriptyline D. Tramadol and gabapentin E. Benzodiazepines and carbamazepine

D. Tramadol and gabapentin - μ-opioid receptors/anticonvulsant gabapentin for neuropathic pain

382. If the muscarinic cholinergic receptor is blocked by atropine, then the nicotinic receptor is blocked by which one of the following drugs? A. Atropine B. Pirenzepine C. Trihexyphenidyl D. Tubocurarine E. Ipratropium

D. Tubocurarine

340. A 25-year old student treated with isoniazid develops peripheral neuropathy and dermatitis: A. Vitamin A B. Vitamin B1 (Thiamine) C. Vitamin B2 (riboflavin) D. Vitamin B6 (pyridoxine) E. Niacin F. Vitamin C G. Vitamin D

D. Vitamin B6 (pyridoxine)

440. Lidocaine was injected into the gingiva to provide regional anesthesia. Blockade of which of the following ion channels most likely mediated the pharmacological effect of the drug in the patient? A. Ligand-gated Ca2+ channels B. Voltage-gated K+ channels C. Ligand-gated Na+ channels D. Voltage-gated Na+ channels E. Ligand-gated K+ channels

D. Voltage-gated Na+ channels (specifically, inactivated ones)

353. Which of the following characteristics of COX inhibitors is the most important for determining its degree of side effects? A. Its half-life B. Whether its reversible or irreversible C. Whether its competitive or non-competitive D. Which isozyme of COX it inhibits E. A and C

D. Which isozyme of COX it inhibits

158. A 7 year old boy is suffering from fever associated with upper respiratory tract infection. A doctor asked by the child's mother to prescribe an antipyretic medication SHOULD NOT order: A. metamizole B. ibuprofen C. acetaminophen D. acetylsalicylic acid E. ibuprofen and acetaminophen

D. acetylsalicylic acid - aspirin

154. A 42-year old woman is treated with unfractionated heparin because of acute pulmonary embolism. Heparin therapy should be monitored with: A. clotting time B. prothrombin time C. INR D. activated partial thromboplastin time (APTT) E. bleeding time

D. activated partial thromboplastin time (APTT)

171. Gancyclovir is: A. an antiinfluenza virus agent B. a nucleoside antiretroviral agent HIV infection C. an integrase inhibitor used for Herpes simplex type 1 infection D. an antiherpes virus agent used for cytomegalovirus infection E. a protease inhibitor used for hepatitis type ?

D. an antiherpes virus agent used for cytomegalovirus infection

610. Oral prednisolone therapy is indicated in: A. heart failure B. hypertension C. idiopathic thrombocytopenic purpura D. anaphylactic shock E. diabetes insipidus

D. anaphylactic shock

367. A 75 yr pt w/ diabetes mellitus type II and concomitant chronic disease of respiratory system requires pharmacological therapy of diabetes (oral or w/ insulin). Which of the following agents should not be prescribed to him? A. glucosidase inhibitor B. short-acting insulin C. thiazolidinodione D. biguanide E. intermediate-acting insulin

D. biguanide (aka Metformin)

146. Torsade de pointes is a specific form of arrhythmia provoked by some antiarrhythmic drugs, which: A. shorten QT interval B. block calcium channels C. block beta-1 receptors D. block potassium channels E. prolonged PQ interval

D. block potassium channels - sotalol class III

613. Which of the following agents is the choice for anaerobic orodental infection: A. doxycycline B. cefuroxime C. tazobactam D. clindamycin E. A and B

D. clindamycin

568. The following inhibit platelet activation and/or aggregation: A. warfarin B. heparin C. thromboxane A2 D. clopidogrel E. enoxaparin

D. clopidogrel

175. A 20-year-old female has been suffering from hirsutism for many years. The most effective treatment of this disease is: A. ethinyl estradiol B. norgestrel C. dydrogesterone D. cyproterone E. norgestimate

D. cyproterone - steroidal antiandrogen

567. In a 45-year-old patient white patches of exudates on oral mucosa were found as the effect of supra-infection. Which of the following agent(s) could be the most likely reason of supra-infection? A. amoxicillin + clavulanic acid B. ceftriaxone C. nystatin D. doxycycline E. A or B or D

D. doxycycline

494. Which of the following antifungal agents is the best choice for therapy of fungal infection of the oral cavity mucosa? A. terbinafine B. griseofulvin C. amorolfine D. fluconazole E. flucytosine

D. fluconazole

164. 72-year-old restless man was treated with agent X, a central acting dopaminergic receptor blocker (D2), which caused sedation and sleepiness. After a few hours impaired consciousness and muscle rigidity had appeared. Examination reveals a "lead pipe rigidity". Agent X is: A. amitriptyline B. diazepam C. selegiline D. haloperidol E. midazolam

D. haloperidol

566. Benzodiazepines differ from barbiturates in that benzodiazepines: A. Facilitate the action of γ-aminobutyric acid (GABA) on neuronal chloride channels B. have anticonvulsant activity C. may induce physical dependence D. have a higher margin of safety than barbiturates E. produce sedation

D. have a higher margin of safety than barbiturates

386. Which of the following clinical situation is the contraindication for digoxin use: A. sinus bradycardia in patients with heart failure B. heart failure in patients with intermittent claudication C. heart failure in patients with a history of angioedema D. heart failure in patients with pre-excitation syndrome E. atrial fibrillation in patients with heart failure

D. heart failure in patients with pre-excitation syndrome

153. Identify the true sentence relating to pharmacokinetics and its consequences in elderly patients? A. in older people, phase I metabolism is intensified which can cause ineffective therapy B. in the elderly patients the reduction of liver flow may cause an impaired metabolism of all drugs C. in elderly patients the dosage of drugs with linear kinetics should be adjusted primarily D. in elderly patient, the dosage of drugs should be adjusted for renal excretion of their active metabolites E. in older people the accumulation of drugs with high affinity for muscle tissue increases

D. in elderly patient, the dosage of drugs should be adjusted for renal excretion of their active metabolites (a person at age 80 have decreased 50% renal excretion compared to someone in their 30s)

19. Which of the following best describes the mechanism of action of benzodiazepines? A. block glutamate receptors in the brain B. activate GABA receptor in the spinal cord C. inhibit GABA transaminase to increase brain levels of GABA D. increase the frequency of opening of chloride ion channels that are coupled to GABA receptor E. stimulate the release of GABA from nerve endings in the brain

D. increase the frequency of opening of chloride ion channels that are coupled to GABA receptor

397. Dihydropyridine derivatives versus verapamil: A. relax smooth muscles in venous bed B. inhibits calcium channels type L C. can induce gastrointestinal reflux D. increased blood flow throughout peripheral arteries and stimulate tachycardia E. decreased heart rate

D. increased blood flow throughout peripheral arteries and stimulate tachycardia (Dihydropyridine)

570. Which of the following effects is responsible for the antihypertensive activity of hydrochlorothiazide? A. dilation of arteries B. decreased preload C. decreased body volume D. increased diuresis E. inhibition of angiotensin activity in cardiovascular system

D. increased diuresis

155. Which of the following effects of dopamine were most likely mediated by the activation of dopamine D receptors? A. increased diuresis B. nausea and vomiting C. constriction of skin vessels D. increased heart rate E. increased stroke volume

D. increased heart rate - because its neurogenic shock, need immediate

28. Which of the following is not true for epinephrine? A. increases heart rate B. increases risk of arrhythmias C. induces inotropic positive effect D. induces dromotropic negative effect E. relaxes bronchi

D. induces dromotropic negative effect

617. The following symptoms • hypotension • delayed, severe dyspnoea and wheezing • fever and chills • aseptic meningitis • cardiomyopathy appearing within 1-2 hours after monoclonal antibody infusion are the symptoms of: A. allergic reaction B. Stevens-Johnson syndrome C. Lyell syndrome D. infusion-related reaction E. red-man syndrome

D. infusion-related reaction

495. Which of the following is the best choice as induction for a 7-year-old child requiring general anaesthesia? A. halothane, enflurane B. sevoflurane, halothane C. lidocaine, propofol D. isoflurane, enflurane E. isoflurane, halothane

D. isoflurane, enflurane

565. A 34 yrs pt was treated w/ H1 antagonist for atopic dermatitis. Which of the following H1 receptor antagonists has the lowest sedative potential and could be considered for pt if he may have to drive a car? A. dimenhydrinate B. diphenhydramine C. promethazine D. loratadine E. hydroxyzine

D. loratadine - no sedation, no CNS entry, Claritin

290. Pt has been suffering from uraemia induced nausea and vomiting and needs an antiemetic agent. Which of the following antiemetics acts through blocking dopaminergic receptors? A. ondansetron B. amitriptiline C. hyoscine D. metoclopramide E. bromocriptine

D. metoclopramide

620. A patient with symptoms of acute pulmonary edema requires therapy with: A. nitroglycerin, furosemide, hydrocortisone B. nitroglycerin, furosemide, morphine C. furosemide, mannitol, theophylline D. morphine, furosemide, captopril E. nitroglycerin, mannitol, theophylline

D. morphine, furosemide, captopril

266. Directly impairs fungal cell membrane? A. terbinafine B. ketoconazole C. ciclopirox D. nystatin E. polimyxin F. praziquantel

D. nystatin

605. Most common mechanism of drug permeation: A. endocytosis B. carrier-mediated transport C. active-transport D. passive diffusion E. none of the above

D. passive diffusion

496. A traveler in a geographical region where chloroquine-resistant P falciparum is endemic used a drug for prophylaxis but nevertheless developed a severe attack of P vivax malaria. The drug taken for chemoprophylaxis was probably: A. atovaquone B. proguanil C. mefloquine D. primaquine E. quinine

D. primaquine

598. The following accelerate healing in gastric ulcers: A. diclofenac B. naproxen C. prednisone D. ranitidine E. acetylsalicylic acid

D. ranitidine

606. Sulfonylureas: A. are used in obese type 1 diabetics B. Induce polyuria and polydipsia C. have been proven to reduce body weight D. require functioning beta cells for a hypoglycaemic effect E. are usually administered subcutaneously

D. require functioning beta cells for a hypoglycaemic effect

16. The best example of depolarising muscle relaxant is: A. atracurium B. pancuronium C. alcuronium D. succinylcholine E. vecuronium

D. succinylcholine

170. Which of the following statements referring to dexamethasone is true? A. It is recommended in substitution therapy for patients who underwent adrenalectomy B. the duration of its activity blocking the hypothalamic- pituitary axis is similar to the one of hydrocortisone C. it exerts the strongest mineralocorticoid effect out of all glucocorticosteroids D. the risk of developing steroid- induced diabetes is higher when taking this drug than when taking hydrocortisone E. The strength of this drug's anti-inflammatory activity is comparable to the strength of prednisone's anti-inflammatory activity

D. the risk of developing steroid-induced diabetes is higher when taking this drug than when taking hydrocortisone

22. Methotrexate can be responsible for which of the following oral conditions? A. edema B. recession C. sensitivity D. ulceration E. gingival hyperplasia

D. ulceration

20. This hypnotic drug facilitates the inhibitory actions of GABA, but it lacks anticonvulsant or muscle relaxing properties: A. clonazepam B. diazepam C. phenobarbital D. zolpidem E. buspirone

D. zolpidem

109. A 55-year old patient working in deratisation business (rodent eradication) got poisoned with a gas of specific smell of bitter almonds. This gas blocks cytochrome oxidase in the cells. This patient should be most probably treated with: A. Protamine B. BAL C. EDTA D. MESNA E. Amyl nitrate

E

179. A man with HIV infection is taking an agent that prevents viral maturation. Which adverse effect is typically associated with this type of drug? A. Anemia B. Pancreatitis C. Neuropsychiatric reactions D. Peripheral neuropathy E. Lipodystrophy

E

199. Which antibiotic should be the agent of 1st choice for patient who requires dental extraction as prophylaxis of endocarditis? A. Aminoglycoside B. III generation cephalosporin C. Clindamycin D. Tetracycline E. Aminopenicillin

E

217. Which of the following is characteristic for thiazides? A. Increased Ca2+ reabsorption in the proximal tubule. B. Activation of the Ca2+/Cl- symporter in distal tubule C. Increased glomerular filtration of Ca2+ D. Decreased renal excretion of vitamin D E. Activation of the Na+/Ca2+ exchanger in the distal tubule

E

297. Bupivacaine has the following features except for: A. Slow onset of action B. Long duration of action C. Indication for intravenous regional anesthesia D. Being a typical representative of amide local anesthetic E. Epinephrine reduces Bupivacaine toxicity

E

304. The indication(s) for clinical use of bromocriptine is/are: A. to prevent lactation B. to treat galactorrhea C. to treat prolactin-secreting pituitary tumors D. In the treatment of parkinsonism and acromegaly E. All mentioned above statements are the proper answer

E

81. A 55-year-old woman with coronary artery disease is treated with drugs which action is characterized by direct effect on vascular smooth muscles, reduced oxygen demand resulting from decreased preload and afterload and reduced late diastolic left ventricular pressure. The most frequent and characteristic undesired effect of treatment with drug X is: A. Peripheral Edema B. Diarrhea C. Gastrointestinal disturbances D. Hypertension E. Headache

E

597. Which of the following is sporicidal concentration of hydrogen peroxide? A. 1% B. 3% C. 0.1% D. 30% E. 13%

E. 13%

474. Diazepam: 1. Is neuroleptic agent 2. Can cause hypertension 3. Is useful for epileptic episodes treatment 4. Never causes fatal overdose A. 1,2,3,4 B. 1,3 C. 1,3,4 D. 2,3,4 E. 3 only

E. 3 only

589. Beta-adrenoceptor antagonists: A. increase cardiac tissue cyclic adenosine monophosphate (cAMP) B. competitively antagonize the β receptor mediated effects of adrenaline and noradrenaline C. salbutamol is an example D. can cause severe orthostatic hypotonia E. A and B are true

E. A and B are true

344. The following antibiotics is/are bactericidal: A. penicillins B. co-trimoxazole C. vancomycin D. erythromycin E. A and C

E. A and C

588. A 1-month old boy contracts an infection that requires antibiotic therapy. With this information alone, which drug below wouldn't you use? A. co-trimoxazole B. erythromycin C. doxycycline D. A and B E. A and C

E. A and C

591. What is true for macrolides therapeutic use in dentistry? A. macrolides are recommended for infections due Gram positive aerobic and lots of anaerobic strains, if amoxicillin is ineffective B. macrolides are recommended for prophylaxis against bacterial endocarditis in susceptible patients who are unable to take medications orally C. macrolides are recommended for prophylaxis against bacterial endocarditis in susceptible patients who are allergic to amoxicillin D. macrolides are recommended for infections due Gram negative aerobic and anaerobic strains, if amoxicillin is ineffective E. A and C are true

E. A and C are true

622. Which of the following is true for volume of distribution? A. is expressed as the amount of drug administered (mg) in relation to the initial plasma concentration (ml) B. a very low volume of distribution may indicate low plasma protein binding of the drug C. a very high volume of distribution may indicate that the drug is negligibly bound to tissue sites D. is expressed as the initial plasma concentration (ml) in relation to the amount of drug administered (mg) E. A and C are true

E. A and C are true

346. An antagonist: A. Has similar binding affinity to the agonist B. Has similar efficacy to the antagonist C. Is always competitive D. Will shift the dose-response curve to the right E. A and D are true

E. A and D are true

500. Pancuronium is: A. a drug depolarizing skeletal muscle, does not cause cramps and muscle tremors B. A drug nondepolarizing skeletal muscles, causes muscle cramps and tremors C. A skeletal muscle and smooth muscle relaxant D. A drug depolarizing skeletal muscle, causes muscle cramps and tremors E. A drug nondepolarizing skeletal muscle, does not cause cramps and muscle tremors

E. A drug nondepolarizing skeletal muscle, does not cause cramps and muscle tremors

621. Plasma protein binding decreases: A. distribution of drug from the plasma to tissue sites B. renal excretion as a result of reduced filtration C. metabolism as a result of diminished uptake by the liver D. A and B are true E. A, B, C are true

E. A, B, C are true

615. Adverse effects associated with the use of theophylline include: A. cardiac dysrhythmias B. convulsions C. oral candidiasis D. nausea and vomiting E. A, B, D are true

E. A, B, D are true

614. Beta 2 agonists: A. relax bronchial smooth muscle B. inhibit release of mast cell and other inflammatory mediators C. examples are: salbutamol, terbutaline, salmeterol D. reduce heart rate E. A, C are true

E. A, C are true

18. What is/are a goal(s) to initiate combined antibiotic treatment? A. to prevent resistance B. to provide synergistic action C. to reduce doses D. A and B E. A,B,C

E. A,B,C

436. Which of the following molecular actions most likely mediated the analgesic effect of morphine? A. Stimulation of substance P release from nerve terminals B. Blockade of K (kappa) receptors C. Stimulation of release of endogenous opioid peptides D. Activation of glutamate receptors E. Activation of u (mu) receptors

E. Activation of u (mu) receptors

47. The proper schedule of chronic pain control (i.g postoperative pain) includes: A. Administration of analgesic as needed B. Administration of analgesic in gradually increasing doses till the therapeutic effect is obtained C. Administration of analgesic at least twice daily D. Administration of analgesic already parenterally E. Administration of analgesic chronically in divided doses, usually every 4-6 hours, not only as needed

E. Administration of analgesic chronically in divided doses, usually every 4-6 hours, not only as needed

364. Major side-effects of antithyroid drugs include: 1. Agranulocytosis 2. Skin rash 3. Hepatotoxicity 4. Vasculitis 5. Arthralgias A. 2 B. 1,2,3 C. 1,3,4 D. 1,3 E. All

E. All

585. Propofol: A. Is given intravenously B. Can cause hypotonia C. Is relatively safety and well tolerated agent D. Produces extremely rapid recovery in patients E. All above are true

E. All above are true

34. Among factors mentioned below find this one which affect hepatic drug metabolism: A. Hepatic blood flow B. Enzyme induction C. Age of patient D. Genetic factors E. All of above

E. All of above

332. M.L. is a 80-year old man who has angina pectoris for 10 years. Anginal attacks are well controlled on isosorbide mononitrate. Among the following adverse effects, which is associated with nitrate therapy? A. Headache B. Tachycardia C. Increased intraocular pressure D. Methemoglobinemia E. All of the above

E. All of the above

48. Which of the following routes of administration could be used to achieve conscious sedation: A. Oral B. Inhalation C. Intravenous D. Rectal E. All of the above

E. All of the above

545. The risk of respiratory depression as a side effect of opioids is increased: A. If opioids are used in combination with general anesthesia B. If opioids .... with alcohol C. If opioids are used .... with asthma D. If opioids are used in high doses ...... ? E. All of the above are correct

E. All of the above are correct

201. Glutaraldehyde: A. Is used as a cold sterilant B. Its activity is not affected by bioburden (blood, saliva) C. Should be used in well-ventilated areas D. Is active against tubercle bacilli, spores, viruses, fungi E. All of the above are true

E. All of the above are true

236. Which one from the following agents provides a patient antianxiety effect? A. Trazodone B. Haloperidol C. Thiopental D. Buspirone E. Alprazolam

E. Alprazolam

85. T.G. has been receiving digoxin for paroxysmal atrial fibrillation and congestive heart failure. After 10 days of digoxin treatment the patient experienced nausea, diarrhea and ventricular arrhythmia. Which of the following agents could be responsible for such symptoms when co-administered with digoxin? A. Acebutolol B. Cholestyramine C. Torsemide D. Verapamil E. Amlodipine

E. Amlodipine

229. Which of the following agents induces the most frequently gingival hyperplasia: A. Losartan B. Quinapril C. Methotrexate D. Metoprolol E. Amlodipine

E. Amlodipine - Gingival hyperplasia: phenytoin, -dipine, cyclosporine *tacrolimus

547. A 27-years-old woman was treated with contains beta-lactam antibiotic. The antibacterial spectrum of activity of this antibiotic covers gram positive community acquired cocci, lot (?) of gram negative community acquired rods. Most likely it is: A. Penicillin G B. Oxacillin C. Penicillin V D. Aztreonam E. Amoxicillin

E. Amoxicillin

326. C.B a 46-year-old man passed coronary artery surgery (Coronary artery bypass graft - CABG) a month ago and now requires also a periodontal procedure. His dentist should prescribe him: A. Clindamycin in the dose of 150 mg to take 4 tablets 2 hours before procedure B. This patients does not require prophylaxis with an antibiotic C. Azithromycin in the does of 500 mg to take 2 tablets 60 minutes before procedure D. Ceftriaxone in the dose of 500 mg to take 4 tablets a day E. Amoxicillin in the dose of 150 mg to take 1 tablet every 8 hours

E. Amoxicillin in the dose of 150 mg to take 1 tablet every 8 hours

555. Clopidogrel is: A. An inhibitor of cyclooxygenase 2 (COX2) B. An antagonist of antithrombin C. An agonist of GP HB/IIa receptors on platelet membrane D. An inhibitor of plasmin activity E. An inhibitor of receptors for ADP on platelet membrane

E. An inhibitor of receptors for ADP on platelet membrane

191. Which from the following does not match: A. Benzodiazepines - paradoxical reaction in elderly B. Gabapentin - treatment of epilepsy C. Barbiturates - respiratory depression D. Tricyclic antidepressants - arrhythmias E. Antipsychotics - increased blood pressure

E. Antipsychotics - increased blood pressure

200. Antiseptics: A. Are used to destroy all forms of microorganisms on inanimate surfaces B. Are used to destroy bacterial endospores on animate surfaces C. Are used to destroy vegetative forms of microorganisms on inanimate surfaces D. Are used to destroy vegetative forms of microorganisms on animate surfaces E. Are used to destroy all forms of microorganisms on animate surfaces

E. Are used to destroy all forms of microorganisms on animate surfaces

531. The mechanism of action connected with the increase of the duration of chloride ion channel openings induced by interaction with GABA receptors is characteristic for: A. Neuroleptics B. Buspirone C. Benzodiazepines D. Tricyclic antidepressants E. Barbiturates

E. Barbiturates

532. Dobutamine is indicated clinically for: A. Hypotension B. Anaphylactic shock C. Thyroid storm D. Tachycardia E. Cardiogenic shock

E. Cardiogenic shock

405. A woman is given a drug that inhibits fungal cell wall synthesis. Which drug is she most likely receiving? A. Griseofulvin B. Fluconazole C. Terbinafine D. Flucytosine E. Caspofungin

E. Caspofungin

534. Which drugs ARE USELESS in PD treatment: A. DOPA Agonists B. MAO-B Inhibitors C. COMT Inhibitors D. L-DOPA E. Cholinomimetic drugs

E. Cholinomimetic drugs

243. A 56-year-old male presents with loose stools containing blood and mucous, abdominal pain and cramps. Diagnosis was pseudomembranous colitis. He was put on antibiotics a week earlier for osteomyelitis. Which of the following antibiotics is most likely to affect his large bowel? A. Tetracycline B. Vancomycin C. Gentamicin D. Ketoconazole E. Clindamycin

E. Clindamycin

118. Patient D.D. was diagnosed for hypertension (comorbid with kidney failure) with furosemide, metoprolol, amlodipine, doxazosin and clonidine. After several months of treatment one of these drugs was discontinued because of depression. The drug shift referred to included: A. Furosemide switched to prazosin B. Metoprolol switched to sotalol C. Amlodipine switched to verapamil D. Doxazosin switched to hydralazine E. Clonidine switched to rilmenidine

E. Clonidine switched to rilmenidine

554. With chronic case of morphine the tolerance for several morphine-related effects develops. Which of the following does not undergo tolerance: A. Nausea B. Analgesia C. Respiratory depression D. Sedation E. Constipation

E. Constipation

188. Certain drug has extremely large volume of distribution. It means that: A. Penetrates only throughout extracellular fluids B. Its volume of distribution is equal to volume of body fluids C. Is hydrophilic D. Contains charges on its surface E. Cumulates inside cells and tissues

E. Cumulates inside cells and tissues

33. Certain drug have extremely large volumes of distribution. It means that: A. Penetrates only throughout extracellular fluids B. Its volume of distribution is equal to volume of body fluids C. Is hydrophilic D. Contains charges on its surface E. Cumulates inside cells and tissues

E. Cumulates inside cells and tissues Note: E is seen mostly, but I saw B also

230. Which of following could induces gingival hyperplasia and hypertrichosis: A. Sirolimus B. Mycophenolate mofetil C. Everolimus D. Azathioprine E. Cyclosporine

E. Cyclosporine

197. A 3-year-old girl was treated with tetracyclines because of severe belleral infection. What is the most likely, irreversible consequence of such therapy? A. Renal impairment B. Aplastic anemia C. Super-intr... D. Myanthema E. Depression of bone growth

E. Depression of bone growth

52. All of the following statements are true EXCEPT: A. Digoxin inhibits K+/NA+APT-ase B. Digoxin is excreted by the kidney C. Digoxin is used in therapy of heart insufficiency D. Digoxin is increase heart contractility E. Digoxin is a drug with a wide range of safety

E. Digoxin is a drug with a wide range of safety (low therapeutic index)

134. A 40-year old male received combination of 3 drugs for hypertension. On control check up his HR was 120/min, whereas before treatment it had been 65/min. Which of the following combinations could not have produced such sinus tachycardia? A. Diuretics, beta-blockers, dihydropyridines B. Angiotensin II receptor antagonist, nitroprusside, beta-blocker C. Diuretics, beta-blockers hydralazine D. ACE inhibitors, beta-blockers, a2-agonists E. Diuretics, hydralazine, ACE inhibitors

E. Diuretics, hydralazine, ACE inhibitors

84. Patient RF complains about exercise induced dyspnoea and ankle edema. On auscultation bilateral pulmonary congestion was revealed, as well as tachycardia and galloping heart rate. Over a few days his urine output has decreased and symptoms exacerbated. Which of the following agents could improve his condition? A. Acetylcholine B. Atropine C. Terbutaline D. Amiodarone E. Dopamine

E. Dopamine

520. The main cause of Parkinson's disease is: A. Excessive metabolism of neurotransmitters B. Peripheral dopamine deficiency C. Age-related blood brain barrier damage D. Peripheral acetylcholine deficiency E. Dopamine deficiency in the CNS

E. Dopamine deficiency in the CNS

113. Drug A is a partial agonist whereas Drug B is a full agonist at the same receptors. A. Drug A less potent. Drug B more effective. B. Drug A less potent. Drug A and B the same effective. C. Drug A more potent. Drug B less effective. D. Drug A less potent. Drug B less effective. E. Drug A more potent. Drug B more effective.

E. Drug A more potent. Drug B more effective.

544. Adding epinephrine to local anesthetics: A. Decrease drug metabolism B. Increase drug potency C. Increase drug distribution D. Improves pKa to pH ratio E. Elongate drug action

E. Elongate drug action

194. 72-year-old female with a long history of anxiety treated with diazepam decides to triple her dose because of increasing fearfulness about "............." Several days after her ...................... Examination, she is found be scuporous and have diminished reaction to pain and decreased reflexes. Her respiratory rate is 8 breaths per minute (BPM) and she has shallow respirations. Which antidote could be given to reverse these findings? A. Naltrexone B. Protamine C. Physostigmine D. Pralidoxime E. Flumazenil

E. Flumazenil

308. The most effective antidote for acute diazepam intoxication is: A. Felbamate B. Flecainide C. Fluoxetine D. Flurbiprofen E. Flumazenil

E. Flumazenil

66. All of the following drugs are used as immunosuppressant for transplant patients EXCEPT: A. Cyclosporine A B. Azathioprine C. Glucocorticosteroids D. Mycophenolate E. GM-CSF

E. GM-CSF

198. Which one of the following effects is unlikely to occur during treatment with tricyclic antidepressants? A. Xerostomia B. Mydriasis C. Alpha receptor blockade D. Urinary retention E. Gingival hyperplasia

E. Gingival hyperplasia

274. The release of insulin from pancreatic beta cells and related hypoglycemia would most likely be stimulated by which of the following? A. Exenatide B. Dapagliflozin C. Metformin D. Sitagliptin E. Glipizide

E. Glipizide

71. The most common adverse effects of cephalosporins included: A. Cardiotoxicity, nephrotoxicity B. Nephrotoxicity, ototoxicity C. Hepatotoxicity, bleeding D. Nephrotoxicity, granulocytopenia E. Granulocytopenia, hypersensitivity

E. Granulocytopenia, hypersensitivity

110. A 36-year old male is seen in the Emergency Department because he can not be aroused from sleep. On examination, he has shallow breathing and pinpoint pupils. Naloxone is administered, and the patient wakes up. He is addicted to: A. Benzodiazepine B. Barbiturate C. Cocaine D. Amphetamine E. Heroin

E. Heroin

206. This drug is given in the dose of 100-500mg i.v. for anaphylactic shock. It concerns: A. Salbutamol B. Clemastine C. Epinephrine D. Diazepam E. Hydrocortisone

E. Hydrocortisone

387. When a patient is treated with a thiazide diuretic for hypertension, all of the following are likely EXCEPT: A. The fall of blood pressure that occurs in the first 2 weeks of therapy results from a decrease of extracellular volume B. The sustained fall in blood pressure that occurs after several weeks of treatment is due to a decrease of intravascular resistance C. After the blood pressure is reduced, hypokalemia remains a complication D. Hyperuricemia may occur E. Hypoglycemia may occur

E. Hypoglycemia may occur

73. Aminoglycosides are bactericidal because: A. Bind to the outer membrane of bacteria changing its permeability B. Inhibit cell wall synthesis C. Compete para-aminobenzoic acid in metabolic pathways D. Bind to ergosterol in cell membrane E. Induce misreading of the mRNA and subsequently impair wall membrane

E. Induce misreading of the mRNA and subsequently impair wall membrane

211. The mechanism of action of fluorides as antibacterial agents is: A. Bacterial cell membrane direct impairment B. DNA structure impairment C. Inhibition protein synthesis D. Inhibition cell wall synthesis E. Inhibition many enzymatic reactions involved in glycolysis and in glucose transport into the cell

E. Inhibition many enzymatic reactions involved in glycolysis and in glucose transport into the cell

75. What is the mechanism of action of macrolides? A. Inhibition of cell wall synthesis B. Disruption of mitotic spindle C. Direct impairment of bacterial cell membranes D. Direct and nonspecific impairment of DNA structure E. Inhibition of bacterial protein synthesis

E. Inhibition of bacterial protein synthesis

487. The mechanism of fluconazole action is: A. Fungal polymerase inhibition B. Fungal Neuraminidase inhibition C. Reverse Transcription Inhibition D. Direct impairment of fungal cell membrane E. Inhibition of ergosterol synthesis

E. Inhibition of ergosterol synthesis

429. Which one of the following statements best describes flumazenil? A. Does not produce withdrawal seizures B. Has the longest elimination half-life C. Is not metabolized into an active metabolite D. Is also used for the treatment of epilepsy E. Is a selective benzodiazepine antagonist

E. Is a selective benzodiazepine antagonist

470. Oxytocin: A. Is a steroid hormone B. Is effective in reducing symptoms of carcinoid syndrome C. Is used for hyperthyricosis treatment D. Commonly causes hypercalcemia E. Is helpful for initiation of lactation

E. Is helpful for initiation of lactation

136. Which of the following statements regarding valproic acid is not true: A. It is used in the treatment of absence seizures B. It may produce hepatotoxicity C. It may raise plasma phenobarbital levels if chronically coadministered D. It potentiates the inhibitory action of GABA in the CNS E. It is linked to a high incidence of gingival hyperplasia

E. It is linked to a high incidence of gingival hyperplasia

322. The second-line agent used in therapy of cardiac arrest related to shockable-rhythm is: A. Amiodarone - 300mg given i.v. B. Norepinephrine - 1mg given i.v. C. Dopamine in infusion - 3μg/kg/min D. Epinerphrine - 1mg given i.v. E. Lidocaine - 100 mg given i.v.

E. Lidocaine - 100 mg given i.v.

271. Which of the following will induce dry mouth in your patient? A. Beta mimetic B. Insulin C. Oral anticoagulant D. Alpha beta-memetic E. Loop diuretics

E. Loop diuretics

427. A drug with very weak opioid activity used in the treatment of diarrhea: A. Oxycodone B. Buprenorphine C. Methadone D. Naloxone E. Loperamide F. Fentanyl G. Tramadol H. Morphine

E. Loperamide

603. In the therapy of Helicobacter pylori infection all of the following drugs are used EXCEPT: A. Amoxicillin B. Metronidazole C. Ranitidine D. Omeprazole E. Magnesium containing antacids

E. Magnesium containing antacids

214. Reflex tachycardia caused by the systemic administration of albuterol can be blocked by: A. Prazosin B. Low-dose epinephrine C. Phenylephrine D. Dobutamine E. Metoprolol

E. Metoprolol

462. Which of the following drug-adverse effects pairs is properly matched? A. Furosemide - brain edema B. Spironolactone - nausea C. Verapamil - tachycardia D. Nitroglycerin - malignant hypertension E. Metoprolol - bradycardia

E. Metoprolol - bradycardia

115. Granisetron is a 5-HT3 receptor antagonist and its main use involves treatment of: A. Peptic ulcer disease B. Motion sickness C. Allergies D. Carcinoid syndrome E. Nausea and vomiting

E. Nausea and vomiting

130. Hepatic coma agent X is recommended to decrease production and absorption of ammonia from: A. Penicillin G B. Tetracycline C. Cephalothin D. Chloramphenicol E. Neomycin

E. Neomycin

477. Which of the following is not antidepressant agent: A. Escitalopram B. Amitriptyline C. Mianserin D. Fluoxetine E. Olanzapine

E. Olanzapine

395. GM-CSF (granulocyte/macrophage colony-stimulating factor) is usually used: A. In autologous bone marrow transplantation B. To shorten the period of neutropenia in patients receiving radiotherapy C. To shorten the period of neutropenia in patients receiving intensive chemotherapy D. All of above are true E. Only answers A and C are true

E. Only answers A and C are true

431. Which of the following correctly describes the intramuscular route of parenteral drug administration? A. Drug absorption is erratic and unpredictable B. Used to administer drug suspensions which are absorbed slowly C. Bypasses the process of drug absorption to achieve an immediate effect D. Cannot be used for drugs that undergo a high degree of first pass metabolism E. Poses more risks than intravenous administration

E. Poses more risks than intravenous administration

516. What is the main difference between amitriptyline and venlafaxine? A. Non-selected mechanism of action B. Clinical uses C. All of the mechanisms above D. Indications E. Postsynaptic receptor activity

E. Postsynaptic receptor activity

129. Patient E.R. is being treated with antiviral drug because of severe influenza. The drug inhibits influenza type A and can be severely neurotoxic. This drug acts based on which of the following mechanisms? A. Preventing production of viral capsid protein B. Preventing virion release C. Inhibiting of reverse transcriptase D. Preventing penetrations of the virus into the host cell E. Preventing uncoating of viral DNA

E. Preventing uncoating of viral DNA

163. Hormonal Replacement Therapy (HRT) usually consists of 17-beta estradiol and? A. Finasteride B. Leuprolide C. Norethindrone D. Danazol E. Progesterone

E. Progesterone

302. Which one of the following IS an selective serotonin reuptake inhibitor SSRI: A. Amitriptyline B. Bupropion C. Desipramine D. Trazodone E. Sertraline

E. Sertraline

78. Which of the following is the most frequent adverse effect of amoxicillin? A. Hepatotoxicity B. Bone marrow depression C. Nephrotoxicity D. Anaphylactic shock E. Skin rashes

E. Skin rashes

213. Which factor does not affect onset and duration of action of local anesthetics? A. pH of tissue B. Nerve morphology C. Concentration of drug D. Lipid solubility of drug E. Speed of injection of local anesthetic

E. Speed of injection of local anesthetic

182. LM is a 21 year-old recently diagnosed with an uncomplicated lower urinary tract infection. Which of the following would be the most appropriate initial therapy for LM? A. Amikacin B. Vancomycin C. Dicloxacillin D. Penicillin V E. TMP/SMX

E. TMP/SMX - (trimethoprim/sulfamethoxazole), inhibitors of folic acid synthesis,can be used to treat uncomplicated UTI

195. It is reasonable to define bioavailability as: A. The fraction of the drug which is soluble at gastric pH B. The percentage of the tablet which is active as opposed to the filler C. The fraction of the administered dose which is absorbed relative to the absorption of a standard form of the drug D. The percentage of an administered dose which is actually absorbed E. The area under the curve (AUC) produced be a single dose of the drug

E. The area under the curve (AUC) produced be a single dose of the drug

49. Which from the following statements concerning oral sedation is false: A. The advantage of this type of sedation is very good patient acceptance and very easy ...... B. The disadvantage of oral sedation is mobility in.....and the potential for a prolonged duration of action C. Patient after oral sedation should not leave the dental office unescorted D. Initial clinical effects of oral sedation are observed approximately 30 mins after ingestion with peak effect at about 60 minutes E. The only determinant of dose of oral sedation is patient's weight

E. The only determinant of dose of oral sedation is patient's weight

311. Which of the following statements about cyclophosphamide is the proper one: A. Cyclophosphamide is inactive until metabolized in the liver by P450 mixed function oxidase B. Aldophosphamide, Cyclophosphamides metabolite, is converted to the cytotoxic molecule, and to acrolein C. Cyclophosphamide has a high affinity for dihydrofolate reductase D. Cyclophosphamide is an analogue to uracil E. The proper answers are A and B

E. The proper answers are A and B

92. Choose the sentence accurately describing the role of leukotriene modifiers in asthma therapy: A. They may be prescribed as controlling monotherapy in severe persistent asthma due to their strong anti-inflammatory effect B. They may be prescribed as controlling monotherapy in mild persistent asthma and as add-on therapy in moderate and severe persistent asthma C. They may be prescribed as short-acting relievers in episodic asthma D. They may be used on as-needed basis for the relief of symptoms due to their strong bronchodilator effect E. They may be prescribed as preventive medicines taken before exposition to exciting factors

E. They may be prescribed as preventive medicines taken before exposition to exciting factors

138. 40 year old patient with diabetes complains of hunger, sweating and palpitations. On examination pale, wet skin and hypothermia were revealed. What is the most likely reason of the symptoms? A. Too low dose of metformin B. Too low dose of insulin C. Glucosuria D. Hyperlipidemia E. Too high dose of sulfonurea

E. Too high dose of sulfonurea

433. Which form of a drug name is most likely known by patients due to the exposure to drug advertisements? A. Nonproprietary name B. FDA approved name C. Chemical name D. Generic name E. Trade name

E. Trade name

114. A patient was admitted to ER with substernal chest pain. Which of the following clinical conditions is the contraindication to beta-blockers: A. Stable angina B. Unstable angina C. Ischemia-induced supraventricular arrhythmias D. Acute myocardial infarction E. Vasospastic angina

E. Vasospastic angina (prinzmetal angina)

434. The following are clinical signs consistent with cocaine intoxication: A. Hypotonia B. Low respiratory rate C. Hypothermia D. Slurred speech E. Ventricular tachycardia

E. Ventricular tachycardia

147. The effect of stimulation of G-protein coupled receptors, that next activate inhibitory subunit G-alpha, changing the activity of adenylate cyclase (AC) is? A. an increase of binding of the receptors to G-protein B. a blockade of ligand binding C. an initiation of conversion of GTP into GDP D. a production of intracellular inositol triphosphate IP3 E. a decrease of cAMP production

E. a decrease of cAMP production

142. Point out a difference in Pharmacokinetics of children as compared to adults: A. permeability of mucus membrane in children is lowered B. bioavailability of fat soluble drugs is increased C. administration of drugs in the rectal route is associated with lower absorption of drugs when compared with oral intake or intramuscular injection D. absorption of intramuscular administration drugs in neonates is better than in adults E. absorption of drugs upon administration on skin is greater

E. absorption of drugs upon administration on skin is greater (true, high absorption of an agent applied to the skin in the neonate and small infants)

599. Omeprazole: A. is an irreversible inhibitor of the hydrogen/potassium adenosine triphosphatase locus of the gastric parietal cell B. reduces gastric acid secretion C. is the drug of choice in Zollinger-Ellison syndrome D. is used for prevention of NSAIDs-induced gastric ulceration E. all above are true

E. all above are true

492. A patient who takes ramipril, furosemide, spironolactone, bisoprolol and digoxin every day the most likely suffers from: A. coronary heart disease B. hypertension C. heart failure D. hypercholesterolemia E. all of above

E. all of above

24. A 27-years-old woman was treated with metronidazole. The antibacterial spectrum of activity of this antibiotic covers: A. Gram positive aerobics B. Gram negative aerobics C. atypical bacteria D. Gram positive aerobes and anaerobes E. anaerobes only

E. anaerobes only

385. Which of the following side effects is not common for angiotensin converting enzyme inhibitors (ACE-I) and for angiotensin receptors antagonists? A. teratogenic effect B. decreased intraglomerular pressure C. hyperkalemia D. first-dose hypotonia E. angioedema

E. angioedema (not widely recognized)

51. A 70 year-old female is treated with sublingual nitroglycerin......of angina. Which of the following is involved in the action of nitroglycerin? A. a-adrenergic activity B. Phosphodiesterase activity C. Phosphorylation of light chains of myosin D. Norepinephrine E. cGMP

E. cGMP

149. 43 year old man, art conservator come to the hospital because of strong spasmodic painful stomach ache for several hours, nausea multiple vomiting. Lab test result normocytic anemia. Significant abdominal tenderness without muscle defence, peristaltic weakness elevated BP 190/100. In last two weeks been working with old iron paint (iron poisoning) and inhalation of dust. Indicate any antidote for this patient: A. atropine B. obidoxime C. disodium-calcium edetate (EDTA) D. pencillamine E. deferoxamine

E. deferoxamine

235. Amlodipine can be responsible for which of the following oral conditions? A. edema B. recession C. sensitivity D. ulceration E. gingival hyperplasia

E. gingival hyperplasia

88. The effects of glucocorticosteroids comprise all of the following, except: A. inhibition of phospholipase A2 B. increase sensitivity of adrenergic receptors C. immunosupression D. inhibition of antibodies production E. hypercalcemia

E. hypercalcemia

27. The effects of glucocorticosteroids comprise all of the following, except: A. inhibition of phospholipase A2 B. increase sensitivity of adrenergic receptors C. immunosupression D. inhibition of antibodies production E. hypercalcemia

E. hypercalcemia - GCs are used in the treatment of hypercalcemia

234. Certain agent causes gingival hyperplasia and hypertension. Which one of the mentioned below? A. interferon alpha B. tacrolimus C. azathioprine D. cyclophosphamide E. mycophenolate

E. mycophenolate

626. Which factor does not affect onset and duration of action of local anaesthetics? A. pH of tissue B. nerve morphology C. concentration of drug D. lipid solubility of drug E. speed of injection of local anaesthetic

E. speed of injection of local anaesthetic

391. Pharmacological treatment, indicated potentially in all patients with symptomatic systolic heart failure, to reduce the risk of premature death include all of the following, except: A. eplerenone B. hydrochlorothiazide C. ramipril D. metoprolol E. spironolactone

E. spironolactone (replaced by eplerenone for symptomatic tachycardia)

586. The combination of two bactericidal agents A and B produces: A. very rarely antagonism B. usually synergism C. usually antagonism D. usually additive effect E. synergism or antagonism depending on the microorganisms susceptibility to drug A and B

E. synergism or antagonism depending on the microorganisms susceptibility to drug A and B

389. This fibrinolytic enzyme is isolated from cultured human kidney cells. It promotes thrombolysis through a direct action on the fibrinolytic system to convert plasminogen into the proteolytic enzyme plasmin. This enzyme is: A. Alteplase B. Anistreplase C. Streptokinase D. Urokinase E. t-PA

E. t-PA

145. Which of the following drugs should be considered as a first choice in the therapy of patients with COPD and is rarely used in the therapy of asthma? A. salbutamol B. fluticasone C. montelukast D. omalizumab E. tiotropium

E. tiotropium - muscarinic receptor agonist (COPD)

366. A 45 yr pt is treated w/ insulin because of type I diabetes. His fasting glycemia is 340 mg/dl, glycemia 2 hr after breakfast (10 am) is 110 mg/dl, 2 hr after lunch (3pm) is 120 mg/dl, 2 hr after dinner (8pm) is 120 mg/dl and at 10 pm is 115 mg/dl. What is the most likely reason of fasting hyperglycemia? A. too high dose of short-acting insulin b4 dinner B. too small dose of short-acting insulin b4 dinner C. too high dose of intermediate-acting insulin b4 breakfast D. too small dose of intermediate-acting insulin b4 lunch E. too high dose of intermediate-acting insulin b4 dinner

E. too high dose of intermediate-acting insulin b4 dinner

596. Simvastatin: A. is an HMG-CoA (3-hydroxy-3-methylglutaryl-CoA) reductase inhibitor B. lowers low-density lipoprotein (LDL) cholesterol C. is particularly useful in hypertriglyceridemia D. acts locally on HMG-CoA reductase in the intestine E. is associated with myositis/myopathy F. A,B,D are true

F. A,B,D are true

1. Which of the following is a phase II metabolism reaction associated with a genetic polymorphism? A. Glucuronidation B. Glutathione C. Conjugation D. Reduction E. Oxidation F. Acetylation

F. Acetylation

338. A 70-year old widower has ecchymoses, perifollicular petechiae and swelling of the gingiva. His diet consists mostly of Cola and hot dogs. A. Vitamin A B. Vitamin B1 (Thiamine) C. Vitamin B2 (riboflavin) D. Vitamin B6 (pyridoxine) E. Niacin F. Vitamin C G. Vitamin D

F. Vitamin C

260. Midazolam is an effective anesthetic because it acts by?

Facilitating GABA-mediated increase in chloride ion conductance

339. Involved in Calcium metabolism: A. Vitamin A B. Vitamin B1 (Thiamine) C. Vitamin B2 (riboflavin) D. Vitamin B6 (pyridoxine) E. Niacin F. Vitamin C G. Vitamin D

G. Vitamin D

261. A 47-year old man recently diagnosed with exertional angina started treatment with sublingual nitroglycerin, as needed and oral isosorbide mononitrate. Which of the following is a potential detrimental effect of these drugs?

Increased cardiac rate

270. Clinically significant post-antibiotic effect:

Is an antibiotic-induced therapeutic effect

481. Ondansetron:

Is effective in preventing emesis and vomiting

263. An μ (mu) opioid receptors antagonist:

Naloxone

262. A 56-year old man felt worse when was in dentistry office. His complains were drug-induced. On examination, his BP was 80/40 mm hg. Heart rate was rhythmical 110/min. Which from the following agents, could be the most likely reason of such his sudden BP decrease?

Nitroglycerin

259. The continued suppression of bacterial growth after an antibiotic has been eliminated from the body is?

Postantibiotic effect

360. CASE: it describes a patient with panic disorder, choose the right drug for it:

SSRI and MAO-inhibitor group of drug is used to treat such condition, familiar all the drug from these 2 groups

257. Which of the following is a sulfonamides-induced oral complications coexisting with severe systemic symptoms?

Stevens-Johnson syndrome (Lyell syndrome)

10. The best example of non-depolarising relaxant is: 1. Atracurium 2. Pancuronium 3. Alcuronium 4. Succinylcholine 5. Vecuronium

1,2,3,5

143. A patient was given I.V. 200 mg of a drug and it is known that 100 mg of this drug was eliminated the first two hours. If the drug elimination follows first order kinetics, how much much will the drug remain in the body after 6 hours?

200/100/50/25 A. -25 mg (Half life calculation)

183. A 33-year old pregnant woman with symptoms of atypical pneumonia requires antibiotic. Unfortunately she is allergic to penicillins and cephalosporins. Which agent seems to be best choice for her? A. Erythromycin B. Doxycycline C. Streptogramin D. Linezolid E. Clindamycin

A

321. Phenolic compounds: A. Are well water-soluble B. Can cause allergies C. Include thymol, hexylresorcinol and alexidine D. Are antifungal agents used topically E. Are first-line agents used against plaque and caries

A

369. Which antibiotic class does not cross the blood-brain barrier? A. Aminoglycosides B. Penicillins C. Sulfonamides D. Fluoroquinolones E. All of the above classes reach therapeutic CSF levels

A

388. A 33-year old pregnant woman with symptoms of atypical pneumonia requires antibiotic. Unfortunately she is allergic to penicillins and cephalosporins. Which agent seems to be best choice for her? A. Erythromycin B. Doxycycline C. Streptogramine D. Linezolid E. Clindamycin

A

524. A drug that exerts its anticonvulsant effects mainly by blocking sodium channels in neuronal membrane is: A. Gabapentin B. Tiagabine C. Clonazepam D. Topiramate E. Ethosuximide

A

581. A patient receiving oral anticoagulant warfarin because of thromboembolism risk needs teeth extraction. The most appropriate management to avoid bleeding complication associated with extraction would be to: A. Stop warfarin administration, 12 hours before surgery B. Administer protamine sulfate in a dose of 30 mg i.v. 2 days before surgery C. Change from warfarin to enoxaparin D. Chance warfarin to acenocoumarol E. Change warfarin to aspirin

A

108. A 55-year-old patient is suffering from renal insufficiency. One year ago she broke her right leg, now she is suffering from compression fracture of the lumbar vertebrae. In prevention of next incidences of fracture she should be treated with: A. 1-alpha hydroxycholecalciferol (1-OHD) B. Cholecalciferol C. Ergocalciferol D. Dihydrotachysterol (DHT) E. Sodium fluoride

A. 1-alpha hydroxycholecalciferol (1-OHD)

224. What is the proper dose and route of administration for epinephrine used for cardiac arrest: A. 1.0 mg i.v. B. 3.0 mg i.v. C. 1.0 mg s.c. D. 10.0 mg i.m. E. 1.0 g i.v.

A. 1.0 mg i.v.

327. This drug belonging to ACE inhibitors is given sublingually to reduce urgently too high blood pressure. The proper dose is: A. 12.5-25 mg B. 0.00125-0.0025 C. 0.125-0.25 mg D. 1.25-2.5 mg E. 125-250 mg

A. 12.5-25 mg

413. Following an overdose of an over-the-counter drug, a young college student has marked gastrointestinal distress and is lethargic and confused, with an elevated body temperature. Lab analysis of blood reveals: pCO2, HCO3-, K+, and an anion gap acidosis. The most likely cause of these signs and symptoms is a toxic dose of: A. Acetylsalicylic acid B. Diphenhydramine C. Pseudoephedrine D. Acetaminophen E. Naproxen

A. Acetylsalicylic acid

370. Which of the following managements is not indicated in cyanide poisoning: A. Activated charcoal administration B. Amyl nitrite administration C. Sodium nitrite injection D. Sodium thiosulfate injection E. 100% oxygen

A. Activated charcoal administration

287. Chlorhexidine affects: A. All bacteria but gram-positive are more susceptible than gram-negative B. Only anaerobes C. All bacteria but gram-negative are more susceptible than gram-positive D. Only gram-negative aerobics E. Only gram-positive aerobics

A. All bacteria but gram-positive are more susceptible than gram-negative

186. Which antibiotics class does not effectively cross the blood brain barrier? A. Aminoglycosides B. Penicillins C. Sulfonamides D. Fluoroquinolones E. All of the above classes achieve therapeutic CSF levels

A. Aminoglycosides (does not achieve therapeutic level in CSF, penetrates poorly to BBB)

399. A 62-year old woman fainted when treated with an antiarrhythmic drug. ECG revealed torsade de pointes. Which of the following drugs may cause this type of arrythmia? A. Amiodarone B. Lidocaine C. Mexiletine D. Metoprolol E. Verapamil

A. Amiodarone

634. of ...... with peptic ulcer disorder disease. All of drugs mentioned below EXCEPT: A. An antidiarrheal agent B. H2 - antagonist C. Proton pump inhibitor D. Prostaglandin analogue E. Coating agent

A. An antidiarrheal agent

323. A 23-year-old man had recurrent or herpes that was effectively treated each time by acyclovir. The patient asked his physician why the treatment was not able to prevent recurrence. Which of the following would be the most appropriate answer for the physician to give? A. Antiviral drugs have no effect on the latent state of viral disease B. Resistance to acyclovir is the rule after one cycle of therapy C. Recurrence is due to a new contact with new source of infection D. Acyclovir has a very short duration of action E. Recurrence is due to hypersensitivity reaction to viral proteins

A. Antiviral drugs have no effect on the latent state of viral disease

190. During his visit in dentistry office 25-year-old patient rapidly developed epileptic seizures. The drug of choice in such situation is: A. Benzodiazepine B. Barbiturate C. Halothane D. Quinidine E. Phenytoin

A. Benzodiazepine

612. Which of the following is used in treatment of hyperprolactinemia? A. Bromocriptine B. Buspirone C. Sumatriptan D. Ondansetron E. Ergometrine

A. Bromocriptine

454. Which of the following is the mechanism of action of dihydropyridines: A. Ca-channels blockade in smooth muscles of arteries B. Ca-channels blockade in central nervous system C. Ca-channels blockade in cardiomyocytes D. Ca-channels distal convoluted tubules E. Ca-channels blockade in smooth muscles of veins

A. Ca-channels blockade in smooth muscles of arteries

319. The most effective antidote for acute mercury intoxication is: A. Calcium disodium edetate (EDTA) B. Deferoxamine C. Dimercaprol (BAL) D. Ethanol E. Penicillamine

A. Calcium disodium edetate (EDTA)

368. The antipsychotic agent which can be useful as an antiemetic (anti-vomiting) is: A. Chlorpromazine B. Doxepin C. Metoclopramide D. Ondansetron E. Thioridazine

A. Chlorpromazine

31. Which of the following drugs wouldn't be used in the treatment of the post-stroke parkinsonism in a 74-year old female? A. Chlorpromazine B. Levodopa C. Selegiline D. Bromocriptine E. Entacapone

A. Chlorpromazine - antipsychotic, induces parkinsonism

12. Hypomineralization of enamel concerning usually the outer third of enamel leading to brown staining and pitting is characteristic feature of: A. Chronic ingestion of excessive amount of fluoride during tooth development B. Chronic ingestion of excessive amount of niacin C. Deficiency of thiamine in elderly D. Excessive use of hydrogen peroxide E. Iron deficiency

A. Chronic ingestion of excessive amount of fluoride during tooth development

631. Chlorhexidine or povidone iodine mouthwash can be beneficial in alleviating adverse effects in a patient who had been treated with: A. Cisplatin B. Roxithromycin C. Digoxin D. Enalapril E. Cefaclor

A. Cisplatin

350. Major route of elimination of inhaled anesthetics is: A. Clearance by the lungs into the expired air B. Metabolism by enzymes of the liver C. Metabolism by the enzymes of the lungs D. Reductive metabolism and excretion by the kidney E. Spontaneous degradation in blood and the lungs

A. Clearance by the lungs into the expired air

4. Which one of the following penetrates bones the best and therefore is recommended for therapy of osteomyelitis? A. Clindamycin B. Clarithromycin C. Doxycycline D. Telithromycin E. Tigecycline

A. Clindamycin

442. Which one of the following drugs inhibits bacterial protein synthesis, preventing the translocation step? A. Clindamycin B. Tetracycline C. Imipenem D. Chloramphenicol E. Gentamicin

A. Clindamycin

95. A 55-year old patient has been receiving a lipid lowering drug that has unpleasant smell and produced flatulence, constipation and abdominal pain. This drug is: A. Colestipol B. Lovastatin C. Niacin D. Fenofibrate E. Probucol

A. Colestipol

137. Patient F.F has total cholesterol level of 300 mg/dl, LDL 130 mg/dl, TG 150 mg/dl and past myocardial infarction about a year ago. The patient takes lovastatin whose administration requires monitoring of: A. Creatine kinase B. Renal function C. Hearing loss D. Monthly complete blood counts E. Heart rate

A. Creatine kinase

335. A 28-year old patient after renal transplant surgery should receive: A. Cyclosporin B. Ethinylestradiol C. Testosterone D. Aldosterone E. Furosemide

A. Cyclosporin

396. The effect of ACE-inhibitors: A. Dilation of veins and arteries, decreased body volume, decreased sympathetic activity, heart remodelling prevention B. Dilation of veins and arteries, decreased body volume, decreased sympathetic activity, heart remodelling prevention C. Decreased Angiotensin II concentration in plasma and increased release of aldosterone D. Dilation of arteries and increased blood pressure E. Increased blood, increased heart rate, hypokalemia

A. Dilation of veins and arteries, decreased body volume, decreased sympathetic activity, heart remodelling prevention

301. Possible mechanisms of carbamazepine involved in bipolar disorder treatment include all of the following EXCEPT for: A. Direct stimulation of serotoninergic 5HT-3 receptors B. Inhibition of uptake and release of norepinephrine from brain synaptosomes C. Inhibition of dopamine receptors supersensitivity development D. Inhibition of GABA-B receptor density E. Indirect inhibition of glutamate activity

A. Direct stimulation of serotoninergic 5HT-3 receptors

372. Patient treated before with the mixture of short acting and intermediate acting insulins given twice daily suffers from severe pneumonia. Which change in insulin therapy is recommended? A. Dose of intermediate-acting insulin should be increased B. Dose of intermediate-acting insulin should be decreased C. Patient should be switched to monotherapy with intermediate-acting insulin given subcutaneously D. Patient should be switched to monotherapy with short-acting insulin given subcutaneously E. Patient should be switched to monotherapy with short-acting insulin given intravenously

A. Dose of intermediate-acting insulin should be increased

383. A 65-year-old woman complained to her physician of acute angioedema. Which of the following drugs was most likely responsible for her problem? A. Enalapril B. Losartan C. Spironolactone D. Nitroprusside E. Diltiazem

A. Enalapril

546. A two-year old boy contracts an infection that requires antibiotic therapy. With this information alone, which drug below would you use? A. Erythromycin B. Doxycycline C. Clindamycin D. Meropenem E. Penicillin G

A. Erythromycin

91. A 50-year old woman suffers from menopause-related flushes. Which agent could be most helpful for her? A. Ethinylestradiol B. Fluticasone C. Mestranol D. Flutamide E. Levonorgestrel

A. Ethinylestradiol

5. Acute toxicity of fluorides involves all of the following except: A. Excessive brown staining and pitting of enamel B. Hypomineralization of enamel C. Degradation matrix proteins D. Nausea, vomiting, diarrhea

A. Excessive brown staining and pitting of enamel

2. For each numbered item select the one lettered option that is most closely associated with it (each lettered option can be selected one, more than once, or not at all). A pure agonist of μ (mu) opioid receptors, a drug with neurotoxic metabolites A. Fentanyl B. Buprenorphine C. Morphine D. Oxycodone E. Tramadol F. Naloxone G. Loperamide H. Methadone

A. Fentanyl

56. Which agents are antihyperlipidemic: A. Fibrates B. Sulfonylureas C. Dexamethasone D. Tiazolodinodiones E. Meglitinides

A. Fibrates

63. Fungal infections are the most frequent oral infections in cancer chemotherapy patients. Which from the following agents are the 1 line prevention and therapy against oral fungal infection? A. Fluconazole B. Griseofulvin C. Flucytosine D. Amorpholines E. Acyclovir

A. Fluconazole

404. A 55-year old patient was put on amphotericin B combined with drug X for cryptococcal infection. Which drug is he most likely receiving as drug X? A. Flucytosine B. Itraconazole C. Terbinafine D. Griseofulvin E. Imiquimode

A. Flucytosine

443. A 45-year-old patient suffers from burning and itching of the lips at the right side and of intraoral pain. He has difficulties in swallowing. His complaints appeared post exposure to the sun and it was the fifth time in his life. On examination vesicles, ulcerations on his lips and intraoral appeared. Which antiviral agent is the first choice for this patient? A. Ganciclovir B. Interferon gamma C. Ribavirin D. Amantadine

A. Ganciclovir

513. . Hyperdopaminergic state as a side effect occurring most markedly in treatment with: A. Haloperidol B. Risperidone C. Olanzapine D. Clozapine E. Quetiapine

A. Haloperidol - Typical antipsychotic. Block dopamine D2 receptor (↑cAMP)

561. This drug inhibits thrombin by accelerating antithrombin III action: A. Heparin B. Ticlopidine C. Warfarin D. Aspirin E. None of the above

A. Heparin

348. The following drugs are used to prevent motion sickness: A. Hyoscine B. Promethazine C. Cinnarizine D. Metoclopramide E. Chlorpromazine

A. Hyoscine (scopolamine)

421. Which side effect is associated with spironolactone? A. Hyperkalemia B. Hirsutism C. Hyperglycemia D. Alkalosis E. Hypercalcemia

A. Hyperkalemia

223. A patient presents the following acute symptoms: confusion, uncontrollable jerking movements of the arms and legs, loss consciousness. Indicate the most possible diagnosis with first-add treatment: A. Hypoglycaemic seizures - glucose B. Anaphylactic shock - norepinephrine C. Asthmatic attack - salbutamol D. Hypertensive crisis - digoxin E. Pulmonary edema - nitroglycerine

A. Hypoglycaemic seizures - glucose

558. Which of the following may be an adverse effect of loop diuretics? A. Hypovolemia B. Arterial hypertension C. Renal failure D. Bradycardia E. Fluid retention

A. Hypovolemia

635. Epinephrine causes: A. Increased heart rate B. Decreased heart rate C. Decreased blood pressure D. Decreased heart contractility E. Decreased oxygen demand

A. Increased heart rate

215. Which of the following molecular actions most likely mediated the positive inotropic action of digoxin? A. Increased release of Ca2+ from the sarcoplasmic reticulum during systole B. Closing of calcium channels in cardiac cell membranes C. Opening of K+ channels in cardiac cell membranes D. Activation of Na+/K+ ATPase E. Activation of the Ca2+/Na+ exchanger in cardiac cell membranes

A. Increased release of Ca2+ from the sarcoplasmic reticulum during systole

428. Which set of three mechanisms of action does topiramate have? A. Increases Na+ channel inactivation, Increases GABA, blocks glutamate B. Decreases Na+ channel inactivation, decreases GABA, blocks glutamate C. Increases Ca+ channel inactivation, Increases GABA, blocks glutamate D. Decreases Ca+ channel inactivation, Increases GABA, blocks glutamate E. Decreases Ca+ channel flow, Increases GABA, blocks glutamate

A. Increases Na+ channel inactivation, Increases GABA, blocks glutamate

579. Atropine sulfate is a drug that: A. Increases conduction velocity in SA node B. Decreases cardiac HR C. Stimulates activity of phospholipase C with increased smooth muscle contraction D. Increases skeletal muscle tone E. Blocks active transport of choline into cholinergic neurons

A. Increases conduction velocity in SA node

526. Atropine sulfate is a drug that: A. Increases conduction velocity in the SA node B. Decreases cardiac heart rate C. Stimulates the activity of phospholipase C with increased smooth muscle contraction D. Increases skeletal muscle tone E. Blocks the active transport of choline into cholinergic neurons

A. Increases conduction velocity in the SA node

60. Hypoglycemia may be the result of ... A. Insulin B. Digoxin C. Alpha-glucosidase inhibitor i.e. acarbose D. Biguanide i.e. metformin E. Thiazolidinodione i.e. rosiglitazone

A. Insulin

415. A 67-year old woman suffered from fatigue and shortness of breath over the past few months. She has diabetes and hypertension for which she has been treated for 25 years with appropriate medications. She is status post three myocardial infarctions (MI x3) and has known inoperable coronary artery disease and CHF. She has been very compliant with her complicated medical regimen, which includes digitalis, an ACE inhibitor (fosinopril), loop diuretics (furosemide), ß-adrenergic receptor blocker (carvedilol) and aldosterone antagonist (spironolactone). On examination, she was noted to be in acute respiratory distress with a respiratory rate of 24, a heart rate of 60, and blood-pressure of 110/60. She was anxious and uncomfortable but polite and cooperative. Neck veins were elevated to 8 cm with the patient partially supine. Lungs revealed rales to the angles of the scapulae bilaterally. Heart revealed a third heart sound and a high pitched holosystolic murmur at the apex consistent with mitral regurgitation. Abdomen was protuberant with a fluid shift consistent with ascites. Extremities revealed 2 to 3+ pretibial pitting edema bilaterally. What can physicians offer this woman? A. Intravenous (cAMP elevating) positive inotropic agents B. Vasodilator therapy with hydralazine C. α-adrenergic blockade with prazosin D. Stop the diuretic, furosemide E. Stop the ACE inhibitor, fosinopril

A. Intravenous (cAMP elevating) positive inotropic agents

225. Which of the following statements best explains why desmopressin was an appropriate therapy of a patient with hemophilia A? A. It increases the plasma levels of factor VIII B. It increases the intestinal absorption of vitamin K C. It inhibits the activity of antithrombin III D. It inhibits the activity of proteins C and S E. It has a pronounced anti-fibrinolytic activity

A. It increases the plasma levels of factor VIII

422. Which one of the following mechanisms of action is characteristic of selegiline, an antidepressant also used in the treatment of Parkinson disease? A. It is a selective MAO-B inhibitor B. It blocks the reuptake of dopamine C. It irreversibly binds to COMT D. It increases release of dopamine vesicles E. It blocks muscarinic cholinergic receptors

A. It is a selective MAO-B inhibitor

41. Which of the following characteristics concerning morphine is incorrect? A. It is used therapeutically to relieve pain caused by severe head injury B. Its action could be reverse by naloxone C. It causes constipation D. It is most effective by parenteral administration E. It causes decrease in blood pressure

A. It is used therapeutically to relieve pain caused by severe head injury

96. Which of the following characteristics concerning morphine is INCORRECT? A. It is used therapeutically to relieve pain caused by severe head injury B. Its action could be reverse by naloxone C. It causes constipation D. It is most effective by parenteral administration E. It causes decrease in blood pressure

A. It is used therapeutically to relieve pain caused by severe head injury

628. .............. cirrhosis of the following local anesthetics, which would potentially ....?? A. Lidocaine B. Benzocaine C. Procaine D. Epinephrine E. Tetracaine

A. Lidocaine

285. Enhancement of the effects of bradykinin is mostly like to occur with drugs like: A. Lisinopril B. Clonidine C. Diazoxide D. Losartan E. Propranolol

A. Lisinopril

256. What is the best form of the therapy of acute, severe hypercalcemia? A. Loop diuretics iv and isotonic saline infusion B. Corticosteroids iv and isotonic saline infusion C. Bisphosphonates iv D. Loop diuretics and bisphosphonates iv E. Thiazides iv and isotonic saline infusion

A. Loop diuretics iv and isotonic saline infusion

460. Blockade of which of the following receptors most likely mediates the following adverse effects: dry mouth, constipation, and blurred vision while reading? A. M3 cholinergic B. 5-HT2 serotonergic C. Alpha-1 adrenergic D. H1 histaminergic E. Nm cholinergic

A. M3 cholinergic

173. A 27-year-old woman was treated with certain antibiotic. Currently she suffers from drug-dependent ulceration in oral cavity and granulocytopenia. This antibiotic is most likely: A. Metronidazole B. Ciprofloxacin C. Amoxicillin D. Doxycycline E. Clindamycin

A. Metronidazole

417. Which of the following is not a good choice for aerobic infection of teeth-related tissues? A. Metronidazole B. Erythromycin C. Amoxicillin D. Cefuroxime E. Azithromycin

A. Metronidazole

79. A 27-year-old woman was treated with certain antibiotic. Currently she suffers from drug-dependent ulceration in oral cavity and granulocytopenia. This antibiotic is most likely: A. Metronidazole B. Ciprofloxacin C. Amoxicillin D. Doxycycline E. Clindamycin

A. Metronidazole

221. Which of the following conditions should be obviously considered to initiate definitive therapy with an antibiotic: A. Microbiological outcomes B. Density of inoculum indicates to the risk antibiotic resistance C. Clinical status of a patient which suggest the most likely antibiotic adverse effects D. Infection origin which indicates to the risk of antibiotic resistance E. Primary site of infection which suggest the most likely microbe responsible for infection

A. Microbiological outcomes

409. An infant with sepsis from E. Coli is treated with an agent that causes nystagmus and vertigo. Which action is exerted by this agent in bacterial cells? A. Misreading of mRNA B. Inhibition of peptide bond formation C. Inhibition of DNA synthesis D. Inhibition of peptide translocation E. Inhibition of amino-acyl tRNA binding to 30S subunit

A. Misreading of mRNA

317. Which mechanism is responsible for atropine pharmacologic effect? A. Muscarinic receptor blockade B. Chloride channel activation C. H2 receptor blockade D. α-adrenoceptor activation E. dopamine D2 receptor blockade

A. Muscarinic receptor blockade

543. Choose false statement - Original and generic preparations: A. Must be therapeutically equivalent B. Must have the same active ingredient C. Must have the same dose of active ingredient D. Must have the same route of administration E. Must have similar PK profile

A. Must be therapeutically equivalent

485. Opioid antagonist used in the treatment of acute overdose: A. Naloxone B. Methadone C. Buprenorphine D. Diphenoxylate E. Meperidine

A. Naloxone

560. The drug of choice in treatment of most tapeworm infection is: A. Niclosamide B. Rifampin C. Nitrofurantoin D. Metronidazole E. Co-trimoxazole

A. Niclosamide

330. H.C. is a 59-year old man who presents to the clinic with unresolved mild chest pain after taking numerous SL nitroglycerin (1 every 5 minutes over a total of 30 minutes). He reports that he has used isosorbide mononitrate 4 times daily for over a year. Which is the most logical reason for the SL tablets' ineffectiveness? A. Nitrate tolerance B. Expired sublingual tablets C. Took too many tablets D. Took the wrong medication E. Nitrate intolerance

A. Nitrate tolerance

309. These antianginal agents dilate arteries and veins simultaneously decrease sympathetic system activity, decrease sodium and water retention. These are: A. Nitrates B. ACE inhibitors C. Dihydropyridines D. Beta-blockers E. Ca-channel antagonists like verapamil

A. Nitrates

549. Your patient with coronary heart disease suffering from acute pain and pressure in his chest requires emergency help. You decide to give him: A. Nitroglycerin 0.5 mg B. Captopril 12.5 mg C. Nifedipine 10 mg D. p....5 mg E. cl...0.75 mg

A. Nitroglycerin 0.5mg

512. The inhalation gas provides good analgesic but weak anesthetic effect: A. Nitrous oxide B. Propofol C. Halothane D. Ketamine E. Thiopental

A. Nitrous oxide - ("laughing gas"), is non-irritating and a potent analgesic but a weak general anesthetic

432. When used in the management of asthma, glucocorticoids are likely to cause: A. Oral thrush B. Excessive salivation C. Oral ulceration D. Vesicles in the oral cavity E. Oral inflammation

A. Oral thrush

424. Which of the following adverse effects is likely to occur in two individuals taking imipramine or chlorpromazine? A. Orthostatic hypotension B. Excessive salivation C. Weight loss D. Pupillary constriction E. Seizure threshold

A. Orthostatic hypotension

467. Chronic administration of corticosteroids .... Cushings syndrome results in? A. Osteoporosis B. Hyperlipidemia C. Orthostatic hypertension D. Dehydration E. Hypercalcemia

A. Osteoporosis

450. A patient is allergic to penicillin, the physician is prescribing another antibiotic. The nurse expects the physician to NOT prescribe: A. Oxacillin B. Doxycycline C. Vancomycin D. Clarithromycin E. Metronidazole

A. Oxacillin

316. Therapeutic effect for ibuprofen or naproxen: A. PGE2 and PGF2a B. Selective inhibit cox2 C. PL2 D. Inhibit PLA2 E. LTB4

A. PGE2 and PGF2a

542. Neostigmine is a drug that we can use to stop action of: A. Pancuronium B. Succinyl CoA C. Fentanyl D. Halothane E. Thiopental

A. Pancuronium - can cause reversal blockade

204. Most drugs gain entry to cells by: A. Passive diffusion with first-order kinetics B. Active transport with zero-order kinetics C. Active transport with first-order kinetics D. Passive diffusion through membrane pores E. Passive diffusion zero-order kinetics

A. Passive diffusion with first-order kinetics

185. AM is a 6-year old female brought to the clinic today because of high fever (39.5°C) and sore throat. On exam, her tonsils are enlarged with bilateral exudate. Her rapid streptococcal antigen test is positive. She has no known allergies. Which of the following drugs would be considered appropriate for first-line treatment? A. Penicillin V - orally x10 days B. Co-trimoxazole - orally x5 days C. Colistin - orally x10 days D. Azithromycin - orally x5 days E. A, B, or C

A. Penicillin V - orally x10 days

563. Which of the following terms best describes a drug that blocks the action of epinephrine at its receptor by occupying those receptors without activating them? A. Pharmacologic antagonist B. Partial agonist C. Physiological antagonist D. Chemical antagonist E. Noncompetitive antagonist

A. Pharmacologic antagonist

564. Two drugs may act on the same tissue or organ through independent receptors, resulting in effects in opposite directions. This is known as: A. Physiological antagonism B. Chemical antagonism C. Competitive antagonism D. Irreversible antagonism E. Dispositional antagonism

A. Physiological antagonism

529. When tested under identical conditions, drug X has following parameters: LD50=50 mg/Kg, ED50=5 mg/Kg. The therapeutic index is: A. Prednisone B. Trastuzumab C. Azathioprine D. Sirolimus E. Cyclosporine

A. Prednisone

279. A 34-year old woman was admitted to the day surgery center for dental operation. Which of the following anesthesia would be the most appropriate for this patient as potently inhibiting nausea and vomit? A. Propofol B. Ketamine C. Fentanyl D. Thiopental E. Haloxane

A. Propofol

482. A 40-year-old patient with rheumatoid arthritis is treated with diclofenac (NSAID). To decrease the risk of NSAID induced peptic ulcer, therapy with X has been ..?. Agent X is: A. Proton pump inhibitor B. B2 receptor antagonist C. Prostaglandin antagonist D. M1 receptor antagonist E. Antacid

A. Proton pump inhibitor

530. The region of the receptor that has a high affinity for the drug and therefore can bind the drug with a chemical bond known as: A. Recognition site B. Effector C. Spare receptor D. G-protein E. Activation region

A. Recognition site

449. A 55-year-old man with diabetes type 1 complained of sudden tremor, palpitations and hunger, during teeth extraction. It happened 2 hours after insulin injection. Which from the following could be the most likely reason for complaints? A. Regular B. NPH C. Lispro D. Glargine E. Ultralente

A. Regular

594. M.H. a 52-years old penicillin-allergic woman with hypertrophic cardiomyopathy requires intra-ligamentary local anesthetic injection. Which of the following prescription was probably made (if at all) by her dentist? A. She does not require prophylaxis with an antibiotic B. Rx Metronidazole 500; Disp 2 tablets; Sig Take 2 tablets 1 hour before procedure C. Rx Clindamycin 150; Disp 4 tablets; Sig Take 4 tablets 1 hour before procedure D. Rx Amoxicillin 500; Disp 4 tablets; Sig Take 4 tablets 1 hour before procedure E. Rx Augmentin 500; Disp 4 tablets; Sig Take 4 tablets 1 hour before procedure

A. She does not require prophylaxis with an antibiotic

226. What is the most likely mechanism of resistance for penicillins? A. Structural modification of PBPs B. Phosphorylation of the drug by bacterial enzymes C. Acetylase production D. Active efflux of the drug from the bacteria E. Methylation of the binding site

A. Structural modification of PBPs

275. A 72 years old man started a treatment that included dabigatran. The inhibition of which of the following molecular actions most likely mediated the therapeutic effect of the drug in this patient? A. Thrombin activity B. Antithrombin III activity C. Plasminogen synthesis D. Prothrombin synthesis E. Thromboxane a2 activity

A. Thrombin activity

514. Which route of administration is not appropriate for systemic effect of a drug: A. Vaginal B. Sublingual C. Transdermal D. Rectal E. Oral

A. Vaginal - for local drug administration

218. In a patient who has an established hypersensitivity to metronidazole, what is the most appropriate drug to use for the management of pseudomembranous colitis? A. Vancomycin B. Ofloxacin C. Doxycycline D. Ampicillin E. Clindamycin

A. Vancomycin

39. Selective alpha-1 adrenolytic causes: A. Vasodilation B. Vasoconstriction C. Increased heart contractility D. Bronchodilation E. Increased heart conductability

A. Vasodilation

423. A patient comes to your office with effort-induced angina and resting tachycardia. You choose the following drug to treat the patient because it slows heart rate by blocking L-type calcium channels in the SA node: A. Verapamil B. Propranolol C. Nitroglycerin D. Isosorbide dinitrate E. Metoprolol

A. Verapamil

286. Which of the following chemotherapeutic drugs inhibits the polymerization of microtubules and can oral ulcerations? A. Vinblastine B. Etoposide C. 5-Fluorouracil D. Cyclophosphamide E. Mercaptopurine

A. Vinblastine

245. The following may cause hypercalcemia is: A. Vitamin D3 B. Bisphosphonates C. Prednisone D. Calcitonin E. Vitamin A

A. Vitamin D3

329. Drug clearance is: A. Volume of plasma cleared of drug per unit of time B. Drug dose cleared from plasma per unit time C. Drug concentration cleared from plasma per unit time D. Volume of blood cleared of drug per unit time E. Volume of urine cleared of drug per unit time

A. Volume of plasma cleared of drug per unit of time

505. Which of the following drugs acts as an agonist of benzodiazepine receptors: A. Zolpidem B. Propranolol C. Buspirone D. Flumazenil E. Olanzapine

A. Zolpidem - Hypnotic agent

159. Indicate the proper empirical treatment for the patients with symptoms characteristic for erysipelas? A. amoxicillin B. gentamicin C. linezolid D. doxycycline E. cotrimoxazole

A. amoxicillin

178. The toxicity of tricyclic antidepressant drugs is mainly due to their A. cardiotoxicity B. myelotoxicity C. haematological disturbances D. ototoxicity E. neurotoxicity

A. cardiotoxicity

624. All of the following are capable of initiating a signal transduction process except: A. combination of an antagonist with its receptor B. combination of an agonist with its receptor C. combination of a neurotransmitter with its receptor D. combination of a hormone with its receptor E. B and C

A. combination of an antagonist with its receptor

375. The most significant pharmacological effect of thiazides and thiazide-like diuretics in therapy of pts w/ primary hypertension depends on: A. decreased fluids volume B. hyperkalemia C. decreased sodium % in smooth mm cells responsible for vasodilatation D. increased calcium % in blood responsible for vasodilatation E. changes in glomerular filtration

A. decreased fluids volume

601. Loperamide: A. decreases intestinal transit time B. increases bulk of gut contents C. is used to prevent nausea D. A and C are true E. A,B,C are true

A. decreases intestinal transit time

598. Which of the following are disadvantages of alcohols used for disinfecting inanimate objects: A. detrimental effect on dental equipment surfaces (leather-like coverings, plastic items) B. ineffectiveness of alcohols against many bacterial spores C. ineffectiveness of alcohols against viruses D. ineffectiveness of alcohols against fungi E. all of above

A. detrimental effect on dental equipment surfaces (leather-like coverings, plastic items)

156. Which of the following drugs, taking into account nephrotoxicity, should be considered as the least safe when chronically used in the older population? A. diclofenac B. tramadol C. fentanyl D. oxycodone E. paracetamol

A. diclofenac - NSAID, causes AKI, hepatotoxic

244. A 56-year-old farmer presented to the clinic with a small, flat, hard swelling under the oral mucosa that formed a fistula discharging yellow granules. One month earlier, he had several teeth extracted because of paradental abscesses, Which of the following drugs would be most appropriate for this patient?

A. erythromycin

29. Bisphosphonates: A. inhibit calcium release from the bones B. inhibit parathormone activity C. increase calcium reabsorption from GI tract D. increase calcium reabsorption from the kidney E. exert some effects on estrogen receptors

A. inhibit calcium release from the bones

174. A.K., a 21 year-old-student treated from anorexia with a drug. Which selectively inhibits reuptake of one of the neurotransmitters in the central nervous system. The clinical uses of this drug are also bulimia and depression. Which of the following is the typical side effect of this drug at the beginning of the treatment? A. insomnia B. sedation C. orthostatic hypotension D. dry mouth E. cardiotoxicity

A. insomnia - SSRI (fluoxetine)

491. The most frequently observed antibiotic resistance: A. is mostly result of genetic material mutation B. for beta-lactam antibiotics is related to high efflux pump activity C. is mostly result of genetic material migration D. for macrolides is related to omitting mechanism activity E. its risk is higher within 2-drugs combination comparing to monotherapy with an antibiotic

A. is mostly result of genetic material mutation

30. Which of the following statements about fluorouracil is true? A. it is an anti-metabolite type anticancer drug B. it is anticancer antibiotic C. it belongs to the group of hormonally active anti-cancer agents D. it is antimitotic agent E. it is an anti-inflammatory drug

A. it is an anti-metabolite type anticancer drug

380. Mark the true statement from the following: A. loop diuretics should not be used alone in the treatment of heart failure B. hyperkalemia is a common adverse effect of thiazides and loop diuretics C. the high-dose thiazides therapy is indicated in severe renal failure (GFR 15ml/min) D. furosemide acts mainly in the distal convoluted tubule E. potassium-sparing diuretics (e.g. spironolactone) are the first choice in the therapy of patients with heart failure and fluid overload

A. loop diuretics should not be used alone in the treatment of heart failure

607. Metformin: A. may cause lactic acidosis B. is particularly useful in alcoholic diabetic patients C. causes severe hypoglycemia D. should be discontinued before major surgery E. is an alpha-glucosidase inhibitor

A. may cause lactic acidosis

152. A patient being poisoned with a certain drug and intoxication symptoms were bradycardia and hypotension. Which of the following could be responsible for: A. metoprolol B. digoxin C. amitriptyline D. atropine E. haloperidol

A. metoprolol - beta blocker used in HTN lowers the BP, lows the HR

618. The following poisons/drugs have been correctly paired with an appropriate antidote/specific measure: A. paracetamol - acetylcysteine B. iron - deferoxamine C. codeine - naloxone D. organophosphorus insecticides - dicobalt edetate E. methanol - ethanol

A. paracetamol - acetylcysteine B. iron - deferoxamine C. codeine - naloxone E. methanol - ethanol

381. 62-year-old women fainted half hour after she received the antiarrhythmic drug. ECG revealed torsade de pointes. Which of the following drugs may cause this arrhythmia: A. quinidine B. lidocaine C. mexiletine D. metoprolol E. verapamil

A. quinidine

502. Pralidoxime chloride is a drug that: A. reactivates cholinesterase that have been inhibited by organophosphate cholinesterase inhibitors B. Reduces the vesicular stores of catecholamines in adrenergic and dopaminergic neurons C. Inhibits the reuptake of biogenic amines into nerve terminals D. Blocks the active transport of choline into cholinergic neurons E. Stimulates the activity of phospholipase C with increased formation of inositol triphosphate

A. reactivates cholinesterase that have been inhibited by organophosphate cholinesterase inhibitors

609. Glucocorticoids: A. reduce inflammation process in the tissues B. increase circulating numbers of T lymphocytes C. increase risk of fungal infections D. are immunostimulators E. aggravate symptoms of oral mucositis

A. reduce inflammation process in the tissues C. increase risk of fungal infections

30. Selegiline's mechanism of action consists in: A. selective inhibition of MOA-B activity B. blocking dopamine reuptake C. irreversible binding to COMT D. intensifying synaptic dopamine release E. blocking cholinergic muscarinic receptors

A. selective inhibition of MOA-B activity (which metabolizes dopamine and increases dopamine levels in the brain, MAO-A not affected unless in high doses of Selegiline is given)

363. A 38 yrs woman in whom CNS malignancy was excluded suffers from unilateral headache accompanied by vomiting and aversion to light. Which of the following agents is routinely used in the treatment of acute migraine attacks and might be useful in this pt? A. sumatriptan B. bromocriptine C. pergolide D. tropisetron E. all of the above

A. sumatriptan - used in migraines

267. Inhibits ergosterol synthesis? A. terbinafine B. ketoconazole C. ciclopirox D. nystatin E. polimyxin F. praziquantel

A. terbinafine B. ketoconazole

162. A patient treated with furosemide because of edema in the course of heart failure loses 1,5 l of fluid each day (deficit=1.5L). What kind of change is necessary to be performed in therapeutic regimen of this patient? A. the dose of the diuretic should be increased B. diuretic from other pharmacological group should be added C. The dose of diuretic should be reduced D. applied diuretic should be changed on other diuretic with different mechanism of action E. changes are not necessary

A. the dose of the diuretic should be increased - N urine output is 2L/day

140. A 54-year-old man was about to receive local anesthesia before a tooth extraction. One year ago the patient underwent an episode of cardiac arrest that was successfully treated with electrical cardio-version. The dentist decided to avoid a local anesthesia with epinephrine because the patient was especially at risk of which of the following drug-induced adverse effects?

A. ventricular tachycardia

269. The major mechanism of HSV resistance to acyclovir is:

The loss of ability to produce viral thymidine kinase

282. Drug metabolism in the elderly is characterized by several characteristic properties: 1. a decreased hepatic metabolism of drugs which biotransformation depends on the liver blood flow 2. a decreased hepatic metabolism of drugs which biotransformation depends on the phase I 3. a decreased hepatic metabolism of drugs which biotransformation depends on the phase II 4. a decreased serum concentration of drugs which are prodrugs and which undergo first pass metabolism

The true statements are: 1,4

468. The following are clinical signs .... with cocaine intoxication:

Ventricular tachycardia

572. A 65-year old patient is treated because of coronary heart disease. One day he fainted because of atrio-ventricular block. Which of the following therapeutic scheme was the most likely reason of his faint?

Verapamil + Metoprolol

125. Phase II (synthetic) reaction include: A. Glucuronidation B. N-acetylation C. O-methylation D. Hydrolysis a. A,B,C b. A,C c. B,D d. D only e. All are correct

a. A,B,C


Kaugnay na mga set ng pag-aaral

English 11 Midterm Review (Terry)

View Set

P3: 1837 Rebellions, Durham Report, Canadian Confederation

View Set

Physics Chapters 32 and 33, Electricity and Magnetism!

View Set

CHAPTER 8 Central Nervous System ANSWERS

View Set

Book Notes: Chapter 3: Types of Muscle Training Part 1

View Set

Chapter 3: Life Policy Riders, Provisions, Options and Exclusions

View Set

Module 7 - Chapter 11 Risk and Return - Biz Finance

View Set